Torts Flashcards
Does D-landowner have a duty to protect a plaintiff not on the defendant’s land from dangers deriving from natural conditions on the defendant’s land?
Under the majority rule, a defendant does not have a duty to protect a plaintiff not on the defendant’s land from dangers deriving from natural conditions on the defendant’s land.
However, a person is required to exercise reasonable care to prevent injury to a plaintiff not on his land resulting from unreasonably dangerous artificial conditions that abut or protrude onto adjacent land, and to protect passersby on a public street from injury deriving from dangerous artificial conditions on defendant’s land.
contributory negligence jurisdiction and common carriers
Common carriers, such as trains and buses, traditionally have a duty to avoid harm to their passengers through the exercise of the highest degree of vigilance, care, and precaution. Thus, as a common carrier, the driver is not subject to the reasonably prudent person test; he is held to a higher standard. Given that he did not properly inspect his bus, he was negligent.
However, if the man was contributorily negligent, he will be barred from recovery in a contributory negligence jurisdiction. Under this approach, a plaintiff is required to exercise due care to protect himself from injury by the defendant. The analysis of whether a plaintiff acted with contributory negligence is similar to the analysis of a defendant’s negligence and considers whether the plaintiff acted as a reasonable person would under similar circumstances. A reasonable person would not have attempted to board a bus wearing roller skates, particularly in the face of posted signs prohibiting the action. As such, in a contributory negligence jurisdiction, the man’s contributory negligence would bar him from recovery.
doctrine of res ipsa loquitur examples
Crashing through the barn while parking the car is the kind of thing that ordinarily does not occur in the absence of a negligent act, and from the known facts, it is more likely than not that the pen pal was responsible. Finally, there is nothing to suggest that the woman had anything to do with the crash, because she was in the passenger’s seat.
The general rule is that there is no affirmative duty to aid a plaintiff. What are the exceptions?
An exception to this general rule is that a person does have a duty to aid a plaintiff if the person’s conduct is responsible for placing the plaintiff in a position of requiring aid.
In some (a minority of) jurisdictions, a defendant is liable for a failure to perform a gratuitous promise to render aid if the plaintiff relied on the promise to the plaintiff’s detriment. In most jurisdictions, however, a defendant who gratuitously promises to take action to help a plaintiff does not have a duty to actually take the promised action.
Special relationship between defendant and plaintiff takes place when the defendant derives economic benefit or when the defendant occupies a position of power over the plaintiff. The following relationships have been generally recognized as triggering the duty to care for the plaintiff:
(1) employer/employee (during and in the scope of employment);
(2) common carrier and innkeeper/customer;
(3) school/pupil;
(4) parent/child; and (
5) jailer/prisoner.
Generally, to recover for negligent infliction of emotional distress, the plaintiff must have been within the zone of danger and must have suffered some physical manifestation of the emotional distress. What is the exception?
There is an exception to these requirements if the defendant negligently mishandles a corpse.
contributory negligence and last clear chance doctrine
In jurisdictions taking a traditional contributory negligence approach, a plaintiff is required to exercise due care to protect himself from injury by defendant. The analysis as to whether a plaintiff was contributorily negligent is similar to the analysis of the defendant’s conduct, in that it considers whether the plaintiff acted as a reasonable person would under the same or similar circumstances. Traditionally, if a plaintiff is found to have engaged in contributorily negligent conduct, he is barred from recovering for defendant’s otherwise negligent conduct.
However, the last clear chance doctrine provides a basis for recovery even where plaintiff has been contributorily negligent. If the injury to plaintiff could still have been avoided through a subsequent exercise of due care by defendant, then defendant is said to have had the last clear chance to avoid harm, and plaintiff’s contributory fault does not bar recovery.
Here, the homeowner took notice of the neighbor’s careless riding, including the fact that his eyes were closed, and the homeowner comprehended the potential risk of harm from the exposed mower blades, yet she nevertheless continued mowing rather than wait until the neighbor passed by. As the homeowner had the last clear chance to avoid the harm, the neighbor may recover for his injuries, even in a contributory negligence jurisdiction.
Is mistake a defense to trespass to land?
No. Trespass only requires the requisite intent to physically enter another’s land. The fact that a defendant is mistaken about where the boundary line is does not negate the intent to enter the property.
i.e. Even though the contractor may have thought he was parking the machinery on the supermarket’s property, the requirement of intent was satisfied.
If a product has a manufacturing defect, strict liability will be imposed on…
…the manufacturer and everyone else in the chain of distribution for the harm caused by the defect. Commercial suppliers at all levels of the distribution chain (i.e., manufacturer, distributor, or retailer) are proper defendants. This is the case even if they did nothing to contribute to the harm.
That the injury is attributable to the defendant is proven by showing that the defect that injured the plaintiff was in existence at the time it left the defendant’s control.
If negligence, rather than strict liability, is the theory of recovery in a products liability action, a defendant will be held liable for a failure to inspect goods for defects if they come from a previously unknown or questionable source.
A defendant that sells a product in a defective condition unreasonably dangerous to the user is …
…strictly liable for any harm done to the consumer or their property. The store is liable because they sold a defective product to the woman and it caused her harm. Even if consumer wasn’t physically injured, she can still recover for damage to her property.
Strict liability of providers of services (vs. sellers of goods)
Generally, providers of services are not held strictly liable for injuries received by their customers. If defective goods are supplied along with services, strict liability is still not applicable, so long as the goods supplied were merely “incidental” to rendition of the services. However, restaurants are frequently regarded as sellers of goods (food) subject to strict liability.
Suppliers of services cannot be strictly liable, but only for negligence.
Generally, a defendant is held strictly liable for injuries resulting from abnormally dangerous activities on his land. Exception?
A defendant will not be liable to a plaintiff who knows of and appreciates the danger justifying the imposition of strict liability and who voluntarily exposes himself to that danger. Such an assumption of the risk will completely bar the plaintiff from recovery.
i.e. the passerby entered the defendant’s land in conscious disregard of the warning signs.
Trespasser and strict liability for his injuries inflicted by a defendant’s animal.
If the plaintiff is a trespasser, strict liability will not be imposed for injuries inflicted by a defendant’s animal while the plaintiff is on the defendant’s land, even as to animals with known dangerous propensities.
If the plaintiff is a known or frequent trespasser, a defendant may only be found liable for negligence.
Here, the rancher knew that locals frequently entered upon a portion of his land to access a neighboring nature reserve. Because the hiker is classified as a frequent trespasser, strict liability will not be imposed.
invasion of his privacy v. defamation
One who publicly discloses private facts of another is subject to liability to the other for invasion of his privacy, if the matter publicized is of a kind that would be highly offensive to a reasonable person, and is not of legitimate concern to the public. Truth is not a defense to publication of private facts. However, unlike defamation, disclosure to a third person is not sufficient publication to garner liability (information must be disseminated to the public).
A strict products liability action requires plaintiff to establish that:
(i) the defendant is a commercial supplier,
(ii) the defendant produced or sold a
defective product,
(iii) the defective product was the actual and proximate cause of the plaintiff’s
injury, and
(iv) the plaintiff suffered damage to person or property.
If the product was dangerous beyond the expectation of the ordinary consumer or a less dangerous alternative or modification was economically feasible, the supplier has supplied a defective product.
Furthermore, while some products may be safe if used as intended, they may involve serious dangers if used in other ways. Courts require suppliers to anticipate reasonably foreseeable uses even if they are misuses of the product.
To hold a manufacturer strictly liable for a defect in a product, the product must have reached the consumer without substantial change in the condition in which it was supplied.
To hold the commercial supplier strictly liable
for a product defect, the product must be expected to, and must in fact, reach the user or consumer without substantial change in the condition in which it is supplied. A product manufacturer is not strictly liable for defects that were not present when the product left the manufacturer’s control.
A manufacturer built a speedboat that it sold through an independent dealership. One day, the boat’s owner was racing the boat dangerously close to shore in a misguided effort to impress some sunbathers on the beach. The owner was heading straight for the beach and then attempted to turn quickly away, but the steering failed and the boat crashed onto shore, injuring a sunbather. The sunbather asserts a claim based on negligence against the manufacturer. At trial, she presents evidence of the above facts and evidence that the steering failure resulted from defect in the boat that was present when it left the manufacturer. At the end of the sunbather’s case, the manufacturer moves for a directed verdict.
How should the court rule?
(A) Grant the motion, because the sunbather
failed to present evidence of negligence on
the part of the manufacturer.
(B) Grant the motion, because the sunbather’s
uncontroverted evidence established that
the boat owner’s negligence was a superseding
cause of the accident.
(C) Deny the motion, because the defect is not
something that would ordinarily occur in
the absence of negligence.
(D) Deny the motion, because the manufacturer
placed a dangerously defective boat into the
stream of commerce.
The court should deny the motion because the sunbather can rely on res ipsa loquitur to get the
case to the jury. To prevail on a negligence claim, the sunbather must show negligent conduct by
the manufacturer, leading to the supplying of a dangerously defective product by the company.
The failure to exercise reasonable care, which is the critical distinction between a products
liability action based on negligence and one based on strict liability, can be established by res ipsa loquitur in a case such as this, because the steering defect at the manufacturing stage would not usually occur without some negligence on the part of the manufacturer.
Hence, (C) is correct and (A) is wrong. (B) is wrong because any negligence on the part of the owner is reasonably foreseeable and will not relieve the manufacturer of the consequences of its negligence. (D) is wrong because simply placing a defective boat into the stream of commerce would present grounds for a strict liability action, but not one for negligence. Some negligence must be shown.
Most courts have refused to extend strict products liability to cases in which plaintiff suffers only economic loss when not accompanied by physical harm to the plaintiff, requiring the plaintiff instead to…
…bring an action for breach of warranty to recover
such damages.
A homeowner purchased a new electric garage
door opener directly from the manufacturer. A
representative of the manufacturer installed the
opener and tested it after installation. The opener
had a safety sensor that was supposed to detect
when anything was in the path of the door and
automatically stop and retract. One day as the
homeowner was backing out of her garage, she
accidently hit the garage door button too early,
causing the garage door to come down and crush
the hood of her car. The homeowner brought a
negligence action against the manufacturer to
recover her damages.
Evidence at trial showed that the manufacturer
had tested a sample of the garage door
openers in the same series as the one installed in
the homeowner’s garage, and had conducted its
normal tests with regard to the installation. The
particular opener installed in the homeowner’s
garage, however, had a defective circuit that
disabled the safety sensor.
To establish a prima facie case, which of the
following best states what the homeowner must
establish?
(A) The safety sensor was defective, and the
manufacturer failed to inspect the safety
sensor on the homeowner’s opener.
(B) The safety sensor was defective.
(C) The safety sensor was defective, and the
manufacturer inspected the safety sensor
but failed to discover the defect.
(D) The safety sensor was defective, and the
manufacturer could have discovered the
defect if it had properly inspected the safety
sensor.
(D) The plaintiff must show that the manufacturer could have discovered the defect. In a product
liability action based on negligence, the plaintiff must show negligent conduct on the part of the
manufacturer that led to the supplying of a product with an unreasonably dangerous defect. The standard of care in a negligence action is reasonable care, and the plaintiff must show that the defendant has failed to exercise such care. The manufacturer’s failure to discover the defect when a proper inspection would have revealed it establishes the failure to exercise due care.
(A) is wrong because it does not refer to the standard of care. The failure to inspect must be shown to be a breach of the duty of care, which would only be the case if the defect could have been discovered by an inspection.
(C) is similarly wrong because it does not show that the manufacturer’s standard of conduct was unreasonable; only if the defect could have been discovered by a reasonable inspection, as indicated by (D), would the failure to do so constitute a breach of duty.
(B) is wrong because it does not refer to the manufacturer’s conduct at all, which is critical to an action based on negligence.
Ten years ago, a chemical manufacturer
decided that it needed a safe place to store toxic
waste that was a byproduct of its manufacturing
processes. A highly reputable engineering
company was hired and an exhaustive survey
was performed to find a site. Once the site
was selected, the chemicals were stored there
underground, using state-of-the-art techniques.
However, the surveyors failed to discover a crack
in the rock of the storage area that was too small
for their instruments to detect. Over a period of
years the chemicals began to seep through the
crack and into the water table below the surface.
Tests disclosed that a nearby city’s water supply,
all of which came from local wells that tapped
into the water table, contained unacceptably high
levels of chemical contaminants. The city was
required to spend $5 million on a complex filtration system to remove the dangerous chemicals from its water.
The city brought an action against the
chemical manufacturer to recover the cost of
building the filtration system. At the end of the
plaintiff’s case establishing the above facts, the
manufacturer moved for a directed verdict.
Should the court grant the manufacturer’s
motion?
(C) No, if the court determines as a matter of
law that storage of the chemicals creates a
foreseeable risk of serious harm even when
reasonable care is exercised by all actors.
(D) No, because the trier of fact could determine
that storage of the chemicals creates a
foreseeable risk of serious harm even when
reasonable care is exercised by all actors.
(C) The court should deny the chemical manufacturer’s motion if it determines that its storage of chemicals was an abnormally dangerous activity, making the manufacturer strictly liable for any harm caused by the chemicals. The city can bring a private nuisance action against the chemical manufacturer for the substantial and unreasonable interference with the use of its water. Nuisances may be based on intent, negligence, or strict liability. A strict liability standard for engaging in an abnormally dangerous activity would apply where the activity (i) creates a foreseeable risk of serious harm even when reasonable care is exercised by all actors; and (ii) is not a matter of common usage in the community. Whether an activity is abnormally dangerous is a question of law that the court can decide on a motion for a directed verdict. Because the city has presented no evidence of negligence on the chemical manufacturer’s part, the only way that the city can survive the manufacturer’s directed verdict motion is if the court finds that a strict liability standard applies, as suggested by (C).
(D) is incorrect because, as discussed above, whether the elements making an activity abnormally dangerous are present is a question of law for the court rather than a question of fact.
Several cars of a freight train transporting
nuclear waste derailed as the train neared a
street crossing. One of the cars struck a motorist’s
car as it was waiting at the crossing gate,
seriously injuring the motorist. The area around
the accident was immediately evacuated, but
fortunately none of the freight cars ruptured in
the derailment.
In an action alleging strict liability against
the railway that operated the freight train,
the motorist established the above facts and
presented evidence of her injuries. The railway
presented evidence that the derailment was
caused by a hidden defect in the spikes that
anchored the rails to the track. The spikes were
manufactured by its regular supplier and had not
previously caused any problems. The railroad
also presented evidence that the local authorities
were supposed to restrict access to roads
crossing the freight line while that particular
train was in transit, but they had failed to do so.
In this action, is the motorist likely to prevail?
Careful! Even when the Q mentions strict liability it’s not necessarily strict liability!
(C) The motorist is not likely to prevail in a strict liability action because her injury did not arise
from the abnormally dangerous propensity of the railway’s activity.
The railway’s transport of nuclear waste likely qualifies as an abnormally dangerous activity because: (i) it creates a foreseeable risk of serious harm even when reasonable care is exercised by all actors; and (ii) the activity is not a matter of common usage in the community. However, the scope of liability extends only
to the dangers that would be anticipated from the activity involved; strict liability does not apply
to harms that were not caused by the normally dangerous propensity of the activity. Here, the
railway’s activity is subject to strict liability because of the danger of radioactivity inherent in
nuclear waste, but not from a derailment by itself. Because the motorist’s injuries were not caused
by the release of radioactivity, strict liability does not apply. The railway would be liable for the
injuries from the derailment only if the motorist established negligence.
A new synthetic liquid was created that could
safely double the output of electrical power
plants. One byproduct of the production of the liquid was a hazardous chemical that was not
biodegradable in the environment. A stateof-
the-art manufacturing plant was built to produce this liquid, and the manufacturing plant secured an expert opinion on how to dispose of the hazardous chemical byproduct. The expert
concluded that the earth beneath the disposal site was impermeable, and that there was no danger of contaminating the underground waters if the chemical were buried. Based on this expert
opinion, the hazardous chemical was buried in
a depression on the land because the head of
the manufacturing plant reasonably believed
that it was safe. The chemical, nonetheless, seeped through the underlying soil strata, and was carried by the flow of percolating water to a neighboring well used by the adjacent sheep
farm to water the sheep. The chemical rendered the water in the well unfit for consumption by
sheep. The sheep farmer had bought the farm after the plant was built. While he was unaware of
the hazardous chemical disposal underground
when he bought the farm, the sheep farmer was
later told his well may be contaminated, and he did nothing about it. The sheep were harmed by drinking from the contaminated well, and the sheep farmer asserts a claim against the manufacturing plant for damages to the sheep in a jurisdiction that follows traditional contributory negligence rules. Which of the following is the manufacturing plant’s best defense?
(A) Many companies converted their power
plants so that they could utilize the synthetic
liquid developed by the manufacturing plant.
(B) The sheep farmer did not do what a reasonable person would have done to prevent
harm to his sheep after he learned that the well was contaminated.
(C) The sheep farmer was contributorily negligent.
(D) The manufacturing plant was in place
and in operation before the sheep farmer purchased his property.
(B) Under the “avoidable consequences” rule, a plaintiff has a duty to mitigate damages to avoid
further injuries from the defendant’s conduct. Because the sheep farmer’s property was damaged in this situation, the sheep farmer’s claim would be based on strict liability. As such, simple contributory negligence would not be a good defense in jurisdictions following traditional contributory negligence rules. (C) is therefore incorrect. But if the plaintiff discovers the existence of the danger and fails to act reasonably to prevent further harm from occurring, the defendant would have a good defense, making (B) the best defense. (A) is incorrect because there is no balancing
of utility and risk where abnormally dangerous activities are involved. (D) is incorrect because
the sheep farmer would have had to have known of and appreciated the risk involved when he
purchased the property to constitute assumption of the risk. Thus, (B) is the only correct answer.
A well-known Oscar-winning producer who
owned the nation’s leading special effects studio
employed a creative designer who was recognized
throughout the industry as one of the top
talents in the special effects field. A competitor
of the producer who wanted to hire the designer
told him that that the producer was in negotiations
to sell his studio to a major Hollywood film
company. In fact this statement was not true, and
the competitor made the statement without any
knowledge of its truth or falsity. The designer,
whose employment contract with the producer
was terminable at will by either party, agreed to
be hired by the competitor. The producer was
very upset when he learned of the competitor’s
action because he knew that the designer would
be almost impossible to replace. The producer
brought suit against the competitor.
If the producer establishes the above facts and
that he suffered damages, which of the following
is likely to be true?
(A) He will recover for intentional interference
with business relations, because the competitor used improper means to hire the designer away from the producer.
(C) He will not recover for intentional interference
with business relations, because the designer was an at-will employee of the producer.
(A) The producer can recover for intentional interference with business relations because the competitor used improper means to hire the designer away.
To establish a prima facie case for interference with contract or prospective economic advantage, the plaintiff must prove:
(i) the existence of a valid contractual relationship between the plaintiff and a third party or a valid business expectancy of the plaintiff;
(ii) the defendant’s knowledge of the expectancy; (iii) intentional interference by the defendant that induces a breach or termination of the relationship or expectancy; and
(iv) damage to plaintiff.
While an interferer’s conduct may be privileged where it is a proper attempt to obtain business for the interferer, not only the ends but also the means of persuasion used must be proper.
Here, the competitor made an intentional misrepresentation with the intent to induce the designer to leave the producer’s employment, and the producer has shown that he suffered actual damage from the designer’s departure.
(C) is incorrect because even though the competitor did not induce the designer to breach
his contract, because it was terminable at will by either party, the competitor did induce a termination of the contractual relationship and interfered with the producer’s business expectancy that the designer would stay in his employ. While the competitor would have been privileged to try to hire the designer away using legitimate means, the privilege does not extend to making intentional misrepresentations about the designer’s current employer.
When a defamatory statement involves a matter of public concern or involves a public figure, the plaintiff must provide competent evidence of…
actual injury (i.e., presumed damages are not permitted absent a showing of knowledge of falsity or reckless disregard of truth). Actual injury is not limited to out-of-pocket loss; it may include impairment of reputation, personal humiliation, and mental anguish.
A man working at a clothing store discovered
that his girlfriend, a coworker, had been taking
money from the cash register. Not wanting to be
a party to the situation, he ended the relationship
and found another job. Not long after this,
the man’s new boss, who knew why the man had
quit, came into the clothing store. He asked the
girlfriend if she missed her boyfriend working
with her at the store. She replied, “Yes, but when
we found that he was stealing from the cash
register, we had no choice but to let him go.”
If the man sues his former girlfriend for
defamation, the fact that the new boss knew the
truth of why the man had left his job at the store
will have what result?
(A) It will act as a complete defense to an action
for defamation.
(B) It will establish that the man has not
suffered any actual injury.
(C) It may diminish the damages that the man
would be entitled to recover.
(D) It proves that the girlfriend had no reasonable
ground for believing that the man was
fired for dishonesty.
(C) The new boss’s knowledge of the true circumstances behind the man’s departure from the store may diminish the man’s recovery.
The girlfriend is liable for defamation because she made a defamatory statement about the man to a third person. As long as it is understood in its defamatory sense, an accusation need not be believed to be actionable. Because the statement that he was stealing at his job constituted slander per se, damages are presumed, completing the prima facie case. Nevertheless, the stated fact will diminish the amount of damages that the man will recover because his reputation was not likely harmed in the eyes of his boss.
(A) is wrong because, as
stated above, the man can establish a prima facie case for defamation even though the defamatory
statement was not believed.
(B) is wrong because actual injury encompasses not only damage to reputation but also humiliation and mental distress, for which the man could recover even if his reputation was not damaged.
(D) is wrong because the fact that the new boss did not believe the statement does not prove lack of basis for the girlfriend to have made it.
The wife of the president of a small but prestigious
private college was also an instructor at
the college. While researching an article about
the college president, a reporter discovered and
revealed in a published news story that while the
wife was dating the president, she had falsified
her academic credentials on her application for a
position with the college. As a result of the news
story, the wife was subject to verbal attacks and
innuendo among her colleagues. She asserted
a cause of action against the newspaper for
defamation and established at trial that the story
about her was not accurate.
What else will the wife need to show to
prevail?
(C) The newspaper published the story with
knowledge that it was false or with reckless
disregard for its truth or falsity.
(D) The newspaper was negligent in publishing
the story.
(D) For the wife to prevail, she will need to show that the newspaper was negligent. Although at
common law defamation liability could be strict, a number of Supreme Court decisions based on the First Amendment now impose a fault requirement in cases involving public figures or
matters of public concern.
A defendant may not be held liable for defamation on a matter of public concern involving a public figure unless, in addition to publishing a false story, it was at least
negligent in ascertaining the truth or falsity of its facts. Here, a story about an instructor at a prestigious college falsifying her academic credentials, while dating the president of the college, likely is a matter of public concern because it affects the perceived integrity of the institution.
(C)
is incorrect because knowledge or reckless disregard is the standard applicable to public figures, and the wife does not qualify as such merely because she is married to someone who may be a public figure.
There are four branches of the tort of invasion of privacy:
(i) appropriation of the
plaintiff’s picture or name for the defendant’s commercial advantage;
(ii) intrusion upon plaintiff’s
affairs or seclusion;
(iii) publication of facts that place plaintiff in a false light; and
(iv)
public disclosure of private facts about plaintiff.
A boy was arrested for shoplifting. His mother
was informed and arrived at the police station,
where a reporter for a daily newspaper took her
picture. The photograph appeared on the front
page of the next day’s edition of the paper, above
a story about an increase in shoplifting arrests.
A caption to the photograph identified her as the
mother of a boy arrested for shoplifting. Later
that week, the mother lost her job as a result of
the story in the paper.
Will the mother recover if she asserts a claim
against the newspaper for invasion of privacy?
(C) No, because her photograph was taken in a
public place.
(D) No, because the caption was accurate.
(C) The facts do not make out a claim for invasion of privacy in any of the four forms that invasion of
privacy takes.
The photograph was not an appropriation for commercial purposes because it was incidental to a legitimate news story and was not used in an advertisement.
The photograph did not involve intrusion because it was taken in a public place.
The news feature did not involve false light because the facts do not suggest that the photograph conveyed a false impression.
Finally, the mother’s appearance at the police station simply was not a private fact, because it is generally agreed that anything visible in a public place may be recorded and given circulation by means of a photograph.
**(D) is incorrect because truth is not a defense to most invasion of privacy actions. Even for false light invasion of privacy, the fact that the caption was true does not preclude recovery if the photograph otherwise conveyed a false impression.** Thus, (C) is a better choice than (D).
A private nuisance action requires a showing that the defendant’s interference with the use and enjoyment of the plaintiff’s property was unreasonable. To be characterized as unreasonable,…
…the severity of the inflicted injury must outweigh the utility of the defendant’s conduct.
***
In balancing these respective interests, courts take into account that every person is entitled to
use his own land in a reasonable way, considering the neighborhood, the values of the respective properties, the cost to the defendant to eliminate the condition complained of, and the social benefits from allowing the condition to continue.
In a private nuisance suit against the record
company, is the fact that the artist owns her
studio and has used it for 15 years controlling?
(B) The artist’s ownership and use of her studio is relevant but not controlling. The fact that one type
of land use was entered into before another is relevant but not conclusive evidence of the reasonableness of the use in a private nuisance action.
***
While not conclusive, conduct consistent with what a zoning ordinance permits is relevant evidence that the use is not a nuisance.
A fire broke out in a home that had been
recently remodeled, destroying the house and
injuring the homeowner. An investigation by the
fire marshal established that the fire started from
a short in some wiring behind a wall. A small
section of wiring that ran to an outlet through a
narrow gap between a furnace chimney and a
hot water pipe had had part of its outer sheath
cut off. The homeowner filed suit against the
electrical company that did the rough wiring.
The parties stipulated for trial that the
company had installed the wiring in compliance
with the blueprints, and that the wiring had been
inspected and approved by the building inspector
before the chimney and the water pipe had been
installed and the walls put up, all by different
contractors. At trial, the homeowner introduced
the report of the fire marshal establishing how
the fire started, and evidence of his medical
expenses and other damages. At the end of the
homeowner’s case, the electrical company’s
attorney rested her case and moved for a directed
verdict. The homeowner’s attorney also moved
for a directed verdict.
How should the court rule on the directed
verdict motions?
(B) Deny the electrical company’s motion and grant the homeowner’s motion for a directed verdict, because the company failed to rebut the presumption of negligence that the homeowner has established.
(C) Deny the homeowner’s motion and grant
the electrical company’s motion for a directed verdict, because the wire could have been damaged by another contractor.
(D) Deny both directed verdict motions, because
the homeowner has presented enough evidence to submit the case to the jury.
(C) The court should grant the electrical company’s motion for a directed verdict because the homeowner has not established a prima facie case of negligence on the company’s part.
The homeowner has established that the electrical company owed a duty to him and that he has suffered harm from the fire caused by the short in the wiring. However, he has not established that the company breached any duty to him. While breach of duty is ordinarily a question for the trier of fact, a plaintiff’s failure to offer any evidence on that element of the prima facie case will permit a directed verdict for the defendant.
Under certain circumstances, the fact that a particular injury occurred may itself establish or tend to establish a breach of duty owed, permitting the trier of fact to infer the defendant’s liability. This is the doctrine of res ipsa loquitur (“the thing speaks for itself”). However, for the doctrine to apply, the plaintiff must show that
(i) the accident causing his injury is the type that would not normally occur unless someone was negligent;
(ii) the negligence was attributable to the defendant; and
(iii) the injury was not attributable to the plaintiff.
The second requirement can often be satisfied by showing that the instrumentality causing the
injury was in the exclusive control of the defendant.
Here, however, the wiring was exposed to work done by other contractors in installing a chimney and a hot water pipe nearby and putting up the walls, and the homeowner has offered no evidence that the cut in the outer sheath of
the wiring was present when the electrical company finished its work. Instead, the fact that the wiring had been approved by the building inspector suggests that the wiring was intact when the electrical company finished. Given these facts, the homeowner has not presented evidence that the negligence was attributable to the defendant. Since res ipsa loquitur does not apply and no other evidence of breach of duty was established, the electrical company’s motion for a directed verdict should be granted.
(B) is also incorrect because the homeowner’s motion for a directed verdict would be denied even if he had established res ipsa loquitur. Establishing res ipsa loquitur merely creates a permissible inference of negligence; it does not create a presumption of negligence. Where the res ipsa loquitur element has been proved, the plaintiff has established a prima facie breach of duty on the defendant’s part and no directed verdict may be given for the defendant. However, it does not require the defendant to present evidence to rebut a presumption. The trier of fact is free to accept the inference of negligence that has been created and find for the plaintiff or reject the inference of negligence and find for the defendant, even if the defendant offers no other evidence on the issue. Thus, the court would not grant the homeowner’s motion for a directed verdict even if he had established res ipsa loquitur.
A bookstore owner entered into an agreement
with a building contractor to have a facade
attached to the front of his bookstore. The
contractor constructed the facade and attached it
to the storefront, using plans prepared by himself
and his own employees. After completing the
work, the contractor was paid the contract price
by the bookstore owner. A week later, a woman
was walking past the front of the bookstore
when the facade and a portion of the original
building collapsed, striking and injuring her.
The woman sued both the contractor and the
bookstore owner for damages arising from her
injuries. The parties stipulated that the attachment of the facade to the storefront caused the building to collapse and that the bookstore
owner was not negligent in selecting or supervising the contractor.
If the woman recovers against the bookstore
owner, does the latter have any right of action
against the contractor?
(A) Yes, because the bookstore owner’s conduct
was not a cause in fact of the injuries
to the woman.
(B) Yes, because the woman recovered from the
bookstore owner on the basis of vicarious
liability.
(C) No, because the bookstore owner selected
the contractor to perform the work.
(B) The bookstore owner has an action against the contractor for indemnification because the
woman’s recovery against the bookstore owner was based on vicarious liability.
While the general rule is that a principal is not vicariously liable for the torts of an independent contractor, a broad exception applies when the duty of care is nondelegable on public policy grounds, such as a landowner’s duty to make his premises safe. Indemnity involves shifting the entire loss between or among tortfeasors, in contrast to apportioning it as in contribution. Where one is vicariously liable for the torts of another, the former has a right of indemnity against the latter.
Here, the bookstore owner was not directly liable to the woman in his capacity as owner of the property because he exercised due care in selecting the contractor, so the judgment against him was on the basis of vicarious liability for any negligence by the contractor, because the bookstore owner’s duty to keep his building safe to passersby was nondelegable.
(A) is incorrect because the bookstore owner, by hiring the contractor, was a cause of fact of the woman’s injuries (i.e., but for the bookstore owner’s hiring of the contractor, the woman would not have been injured). However, that does not address the theory on which the woman recovered against the bookstore owner.
Each member of the partnership is …. liable for the tortious conduct of another partner committed in the scope of the partnership’s affairs.
vicariously
Independent contractor
The general rule is that a principal is not liable for tortious acts of an agent who is an independent contractor.
An agent is likely to be an independent contractor if she:
(i) is engaged in a distinct business of her own;
(ii) controls the manner and method by which she performs her tasks;
(iii) is hired to do a particular job;
(iv) supplies her own tools and materials;
(v) is paid a given amount for the job; and
(vi) is hired to do a short-term, specific job.
Despite the general rule, a principal can be held liable for the tortious acts of an independent contractor if:
(i) the independent contractor is engaged in inherently dangerous activities; or
(ii) the principal has a duty that is nondelegable on public policy grounds (e.g., a land occupier’s duty to keep his land safe for business invitees).
Also, a principal can be held liable for his own negligence in selecting an incompetent independent contractor.
The common law rule is that an automobile owner is (not) liable for torts committed by another
person driving the automobile.
not liable
By statute or judicial precedent, many states have imposed liability on a car owner for the tortious conduct of an immediate family member driving the car (family car rule) or for the tortious conduct of anyone driving the car with consent (permissive use rule).
A developer constructed several small stores
in a commercial district. She received a bid
from a contractor to install awnings on the
front windows of the stores. The developer had
heard that the contractor did shoddy work, but
the price was right and the contractor expressly
assumed all of the risk of any liability. The
developer subsequently sold one of the stores to a barber. A few months later, an awning collapsed
without warning, injuring a customer who was
about to enter the barbershop. An investigation by the building inspector revealed that the awning collapsed because the brackets used by the contractor were cheaper and weaker than the required brackets, although they looked the same. The developer and the contractor are now both bankrupt. If the customer sues the barber for his injuries, is the customer likely to prevail?
(A) Yes, because the barber had a nondelegable
duty to keep the premises safe for customers
and those passing by.
(D) No, because the barber had no opportunity
to oversee the contractor’s actions.
(D) The customer is not likely to prevail because the barber had no opportunity to oversee the
contractor’s work.
A property owner owes a duty to those adjacent to the premises to take due precautions to protect them from dangerous conditions, and a business owes its customers a duty to make reasonable inspections to discover and make safe any dangerous conditions. Further, that duty cannot be delegated to an independent contractor; the owner remains vicariously liable.
However, nothing in the facts establishes that the barber breached his duty to the customer. There was nothing wrong with the brackets evident from a reasonable inspection, and the awning collapsed without warning. Further, the barber was not involved in hiring or supervising the contractor and would not be responsible for the contractor’s negligence.
Hence, because no facts point to negligence by the barber, the customer is not likely to prevail.
(A) is incorrect even though it is true that a business owner would be vicariously liable to customers and passersby injured by the negligent work of an independent contractor that he hired. Here, however, the barber did not
engage the contractor and is not responsible for the contractor’s conduct.
A delivery company employed several messengers to deliver packages by car to nearby towns. The company also allowed some employees to use company cars for personal use from time to time. A clerical employee had her car in a body shop because she had run a red light and been broadsided by another vehicle. This was the second time she had run a light and been hit. She borrowed a company car for the weekend and was using it to do some grocery shopping. The employee negligently went through a red light and crossed the path of a rented van. The man driving the van swerved
to avoid the employee and struck a light post and
several parked cars, severely damaging the van.
At the time of the accident, the driver of the van
was exceeding the posted speed limit; he would
have been able to avoid hitting the light post and
the cars had he been going the proper speed.
The leasing company that had rented the van to the driver brought a lawsuit against the delivery company employee and the delivery company. The jurisdiction retains traditional contributory negligence rules. If the delivery company prevails in the lawsuit, what is the most likely reason?
(A) The delivery company had no reason to
know that its employee had a poor driving
record.
(B) The driver of the van had the last clear
chance to avoid the accident.
(A) If the delivery company prevails, it will be because it entrusted its car to its employee without having reason to know that she had a poor driving record.
In the absence of negligence on the delivery company’s part, it will not be liable for its employee’s negligent driving either as her
employer or as the owner of the automobile she was driving.
An employer will be vicariously liable for tortious acts committed by its employee only if the tortious acts occur within the scope of the employment relationship.
Here, although the employee was using the company car, she was not conducting any business for the delivery company. Her use of the car to go grocery shopping was a personal errand outside the scope of her employment for which the delivery company is not vicariously liable.
The delivery company is also not vicariously liable for permitting its employee to drive its car—the general rule absent a statute to the contrary is that an automobile owner is not vicariously liable for the tortious conduct of another driving the owner’s automobile.
However, the owner may be liable for its own negligence in entrusting the car to a particular driver. If the delivery company knew or should have known that its employee had a poor driving record, its furnishing the employee with a car would constitute a breach of its duty to other drivers. However, if it is determined that the delivery company had no reason to know of the employee’s poor driving record, it will not be liable.
(B) is incorrect because the fact that the driver of the van had the last opportunity to avoid the accident is irrelevant to the leasing company’s right to recover from the delivery company. The doctrine of last clear chance does not apply because it is essentially plaintiff’s rebuttal against the defense of contributory negligence; it would not be raised by the delivery company as a defense (i.e., if the delivery company asserted that the driver of the van was contributorily negligent, the leasing company could rebut by asserting that the delivery
company employee had the last clear chance to avoid the accident).
A statute requires that any pilot who flies passengers for hire must have a commercial pilot’s license. An experienced pilot who had only a private pilot’s license and not the
commercial license required by statute was asked by an attorney to fly her to another city to close a deal. The attorney knew that the pilot did not have a commercial license but the only commercial flight to the city was at an inconvenient time. The pilot flew the attorney through bad weather and landed safely, but because of a minor navigational error he landed at an airport a few miles away from the airport he was heading for. As he was going to start taxiing toward the hangar, another plane struck the aircraft. The student pilot of that plane had ignored the control tower’s instructions and gone onto the landing runway instead of the takeoff runway. The attorney was injured in the collision. If the attorney sues the pilot for her injuries,
who will prevail?
(A) The pilot, because the attorney knew he lacked a commercial license and voluntarily assumed the risk of flying with him.
(B) The pilot, because the injuries to the attorney were caused by the negligence of
the student pilot of the other plane.
(C) The attorney, because the pilot violated a statute designed to prevent persons without commercial licenses from flying passengers for a fee, and such violation imposes liability per se.
(D) The attorney, because the pilot landed at the wrong airport, and but for this mistake the attorney could not have been injured by the other aircraft.
(B) The pilot will prevail because the conduct of the other plane’s student pilot constituted a superseding intervening force that relieves the pilot from liability.
To establish a prima facie case for negligence, the attorney must show that the pilot’s breach of his duty to her was the actual and proximate cause of her injury.
The attorney can establish actual cause because but for the pilot’s error, she would not have been injured. However, not all injuries “actually” caused by a defendant
will be deemed to have been proximately caused by his acts. The general rule of proximate cause is that the defendant is liable for all harmful results that are the normal incidents of and within the increased risk caused by his acts. This rule applies to cases such as this, where an intervening force comes into motion after the defendant’s negligent act and combines with it to cause plaintiff’s injury (indirect cause cases).
Here, the pilot’s navigational error did create a greater risk of collision with other planes in the process of landing, but it did not increase the risk of a plane
using the landing runway to take off in disregard of the control tower’s instructions once the pilot was safely on the ground. Hence, the student pilot’s unforeseeable conduct was not within the increased risk created by the pilot’s negligence and constitutes a superseding force that breaks the causal connection between the pilot’s conduct and the attorney’s injury, enabling the pilot to avoid liability to the attorney.
(A) is incorrect because assumption of the risk requires knowledge of the
specific risk and the voluntary assumption of that risk.Although the attorney knew that the pilot lacked a commercial license, she also was under the impression that he was a very good pilot. There is no indication that she knew of or voluntarily assumed any risk. Certainly, she did not assume the risk of the type of harm she suffered.
The plaintiff was exiting from a parking
garage owned and operated by the city when he
discovered that the exit ramp was blocked by
construction barricades and a pile of brokenup
concrete. No workers or detour signs were
around and the plaintiff was in a hurry, so he
backed up and drove down an entrance ramp that
was clearly marked as such. As he came around
a corner, his car was broadsided by a pickup
truck. The plaintiff was seriously injured in the
collision. A statute in the jurisdiction requires
drivers to obey all traffic directional markings in
both public and private parking lots and garages.
The jurisdiction retains governmental immunity
for municipalities.
If the plaintiff brings a lawsuit against the
city to recover for his injuries, which of the
following facts will be LEAST helpful in the
city’s defense?
(A) The plaintiff was aware that another exit on
the other side of the garage was open.
(B) The construction workers responsible for
blocking off the exit ramp were employees
of an independent contractor rather than the
city.
(C) The city does not collect fees or make a
profit in the operation of the garage.
(D) The pickup truck driver could have avoided
the plaintiff but recognized him as an old
enemy and deliberately ran into him.
(B) The fact least helpful to the city’s defense of the plaintiff’s lawsuit is the identity of the workers
who blocked the exit ramp.
Under vicarious liability rules, a principal will be liable for the tortious acts of an independent contractor if the duty is nondelegable on public policy grounds; included is the duty of a possessor of land to keep its premises safe for its invitees. If the workers were negligent in leaving the ramp blocked without providing another means of exiting, the fact that they were not city employees would not absolve the city of liability; hence, their identity would be of no help to the city’s defense.
(A) is incorrect because if the plaintiff was aware of an alternate route, he may have been contributorily negligent in exiting down the entrance ramp. A plaintiff’s contributory negligence may be established by violation of an applicable statute. However, as with a statutory duty imposed on a defendant, the plaintiff’s violation of the statute may be excused if compliance was beyond the plaintiff’s control. If no other means of exiting the garage were known to the plaintiff, he may be excused for violating the traffic statute; however, if he knew of an alternative exit, the city will probably be able to establish contributory negligence on his part by his violation of the statute, reducing his potential recovery.
(C) is incorrect because whether the city
collects fees and makes a profit in operation of the garage will be considered by the court in determining whether the jurisdiction’s governmental immunity applies.
Where municipal immunity still exists, courts have limited its scope by differentiating between “governmental” and “proprietary” functions of the municipality. If the municipality is performing a function that might as well have been provided by a private corporation, the function may be construed as a proprietary one and no immunity will attach. The inference that a function is proprietary will be strengthened
where the city collects revenues by virtue of providing the service.
Hence, the fact that the city is not collecting revenues or making a profit in operating the garage will make it less likely that the function will be deemed to be proprietary and more likely that it will be deemed to be governmental
and thus immune; in other words, it will be more helpful rather than less helpful in the city’s defense.
(D) is incorrect because the pickup truck driver’s conduct under these circumstances
would be deemed a superseding force that breaks the causal connection between any negligence on the part of the city and the plaintiff’s injury. Assuming that the city workers were negligent, the fact that an independent intervening force caused the injury generally would not cut off the city’s liability, because its negligence created a foreseeable risk of that harm occurring. However, where this foreseeable harm is caused by an unforeseeable crime or intentional tort of a third party, most courts would not hold the city liable, treating the crime or tort as a superseding force.
Here, while blocking the exit ramp created a foreseeable risk that someone might collide with the plaintiff, it was not foreseeable that his enemy would take that opportunity to commit an intentional tort against him. Because the pickup truck driver’s conduct was unforeseeable under the circumstances in choice (D), the city would be relieved of liability for any negligence in blocking the ramp.
On the way home from a nightclub, a passenger began yelling at the designated driver claiming that he was not taking the best route back to her house. The driver disagreed and contended that his route was the quickest. The passenger impulsively grabbed the steering wheel, causing the car to swerve and strike a pedestrian, injuring him. At trial, the pedestrian established that the driver’s license had expired the day before the accident. The driver’s traffic record qualified him for an automatic renewal of his driver’s license, but he had forgotten to submit it in time. A statute in the jurisdiction makes it an offense to drive a vehicle on any public road in the state without a valid driver’s license.
Will the pedestrian prevail?
(A) Yes, because the driver violated a statute by
driving without a valid license.
(B) Yes, because the driver failed to control his
passenger.
(C) No, because the driver did not start the
argument.
(D) No, because the passenger’s action was the
proximate cause of the injury.
(D) The pedestrian will not prevail because the passenger’s grabbing of the wheel is the negligent conduct that caused the pedestrian’s injuries.
To establish a prima facie case for negligence, the following elements must be proved:
(i) the existence of a duty on the part of the defendant to conform to a specific standard of conduct for the protection of the plaintiff against an unreasonable risk of injury;
(ii) breach of that duty by the defendant;
(iii) the breach of the duty by defendant was the actual and proximate cause of plaintiff’s injury; and
(iv) damage to plaintiff’s person or property.
Here, the driver’s actions were an actual cause of the pedestrian’s injury because, but for the driver’s driving and the passenger’s grabbing the steering wheel, the injury to the pedestrian would not have happened. However, the driver’s actions were not a proximate cause of the injury because the passenger’s grabbing of the steering wheel was a superseding intervening force. A superseding force is one that serves to break the causal connection between the initial wrongful act and the ultimate injury, and itself becomes a direct immediate cause of such injury. Thus, the first actor would be relieved of liability from the consequences of his antecedent conduct. The passenger’s conduct in suddenly grabbing the steering wheel was an unforeseeable intervening force creating an unforeseeable harmful result, and thus constituted a superseding force. Consequently, the driver would be relieved of any negligence liability since the passenger’s actions were the proximate cause of the accident.
(A) is incorrect. A clearly stated specific
duty imposed by a statute may replace the more general common law duty of due care when (i)
the plaintiff is within the class to be protected by the statute, and (ii) the statute was designed to
prevent the type of harm suffered. The statute probably does not apply here because it is intended to keep unsafe drivers off the streets, and there is no indication that the driver is an unsafe driver, or that any driver could have prevented the injury when the passenger grabbed the steering wheel. Even if the statutory standard were applicable, a violation means only that plaintiff will have established a conclusive presumption of duty and breach of duty. It does not, however, establish causation or damages. Here, the fact that the driver does not have a valid license is not the proximate cause of the pedestrian’s injury, as discussed above.
(B) is incorrect because the driver had no way of knowing that the passenger would grab the steering wheel. The driver’s conduct will be measured against that of the ordinary, prudent, reasonable person who drives a vehicle. An
ordinary, prudent, reasonable person would not have foreseen that one of his passengers would
impulsively grab the steering wheel, and therefore there are no special safety precautions that the driver should have taken as part of his duty of care toward pedestrians.
(C) is incorrect because, even if the driver had started the argument, this would not justify the passenger’s grabbing of the steering wheel. In either case, the passenger’s actions rather than the driver’s would be considered the proximate cause of the pedestrian’s injuries.
A doorman negligently locked a door that an office worker was intending to use to exit an office building, so the worker was forced
to use a different exit. As she stepped onto the
sidewalk outside the building, a car careened out
of control on the street and jumped the curb. The
car struck and injured the worker and then drove
off. The driver was not found. The worker brought suit against the doorman, seeking damages for her injuries. At trial, the parties stipulated that the doorman was negligent in locking the door and that the worker suffered injuries when she was struck by the car. The worker also established that if she had exited from the door she was intending to, she would not have been struck by the car. At the end of the worker’s case, the doorman moved for a directed verdict in his favor.
How should the judge rule?
(A) Grant the motion, because the driver of the
car was the actual cause of the worker’s injuries.
(B) Grant the motion, because the car was an
unforeseeable intervening force.
(C) Deny the motion, because the jury could
find that but for the doorman’s negligence, the worker would not have been injured.
(D) Deny the motion, because the jury could find that the doorman’s negligence was a foreseeable concurring cause of the worker’s injury.
(B) The court should grant the motion because the evidence establishes that the car was a superseding force that cut off the doorman’s liability for his negligence under proximate cause principles.
The general rule of proximate cause is that the defendant is liable for all harmful results that are the normal incidents of, and within the increased risk caused by, his acts.
An indirect cause case is one where the facts indicate that a force came into motion after the time of defendant’s negligent act and combined with the negligent act to cause injury to the plaintiff. Whether an intervening force will cut off the defendant’s liability for the plaintiff’s injury and be deemed superseding is determined by foreseeability.
Here, nothing in the facts suggests that a car jumping the curb was a foreseeable consequence of the doorman’s negligently locking the door. Hence, the judge should grant the motion because the worker has failed to establish the proximate cause element of
his prima facie case.
(A) is wrong because the doorman was also an actual cause of the worker’s injuries—but for the doorman’s negligence, the worker would not have been on the sidewalk where the car jumped the curb.
(C) is wrong because it establishes only actual cause. A directed verdict is appropriate because no evidence establishes the proximate cause element of the worker’s case.
(D) is wrong because the facts do not establish foreseeability. While the doorman’s negligence
was a concurring actual cause of the worker’s injury, it was not a proximate cause because
the injury that occurred was unforeseeable.
A homeowner raked up numerous dead, dry
leaves that had fallen on his yard and set fire
to the pile, even though the wind was blowing
at 15-20 miles per hour that day, and a county
ordinance made it unlawful to burn leaves on
any day when the wind speed exceeded 10
miles per hour. After the homeowner went
into his house, the wind whipped the burning
leaves into the air and deposited some of them,
still burning, on the wood-shingled roof of a
neighboring house, setting the house on fire.
A driver was passing by and saw the burning
roof of the neighbor’s house. He pulled over, got
out of his car and ran across the street toward
the neighbor’s yard so that he could warn the
occupants of the house. The driver was struck by
a paramedic’s vehicle just arriving in response to
a telephone report of the fire. The driver brought
an action against the homeowner to recover for
the injuries he suffered from being struck by the
emergency vehicle.
How should the court rule in this action?
(A) For the driver, because the homeowner’s
negligence was a proximate cause of the
driver’s injuries.
(C) For the homeowner, because the emergency
vehicle was the actual cause of the driver’s
injuries.
(A) The court should rule for the driver. The homeowner’s negligence in burning leaves during a period of relatively high winds resulted in a fire, and it is foreseeable that third persons will attempt to rescue the victims of the tortfeasor’s negligent acts. To the extent that the emergency vehicle was an intervening force, it was a normal response (responding to the fire) and was not unforeseeable. Thus, the homeowner’s negligence was a proximate cause of the driver’s injuries.
(C) is incorrect. The homeowner’s negligence was a cause in fact of the driver’s injuries, because it was the homeowner’s negligence which both prompted the driver to attempt a rescue of the occupants of the burning residence, and brought the emergency vehicle onto the scene. It is possible for the separate actions of two independently operating actors to combine to injure the plaintiff, and whether either of them is liable for those injuries depends on the elements of the negligence analysis, including proximate cause. Here, the homeowner breached the duty of due care, and that breach was an actual and a proximate cause of the injury to the driver.
A skier broke his leg when he was knocked
down by the chair lift as he tried to avoid other
skiers who had fallen off while disembarking.
The ski resort employee operating the lift had
not been paying attention and had failed to stop
the lift. Ski patrol personnel placed the skier
on a stretcher, which they then hooked up to a
snowmobile to bring him down the mountain.
The route down ran along the edge of a ski trail.
Midway down, a novice snowboarder tried to
see how close he could come to the stretcher
without hitting it, but he lost control and landed
on top of the skier’s leg, damaging it further.
The skier filed a lawsuit against the snowboarder
and the resort in a jurisdiction that has adopted
a comparative contribution system in joint and
several liability cases. At trial, the skier’s physician testified that the skier’s leg was permanently disabled, but that neither injury, by itself, would have caused the permanent disability and it was impossible to quantify how much each injury contributed to the disability. The jury determined that the damages from the permanent disability equaled $2 million, and that the snowboarder and the resort were each 50% at fault. What amount of damages can the skier
recover from the snowboarder for his permanent
disability?
(A) $1 million, because the jurisdiction follows
comparative contribution rules.
(B) $2 million, because it was not possible to
identify the portion of the injury that the
snowboarder caused.
(B) The skier can recover $2 million from the snowboarder because the snowboarder is jointly and severally liable for the injury.
The doctrine of joint and several liability provides that when two or more tortious acts combine to proximately cause an indivisible injury to plaintiff, each tortfeasor will be jointly and severally liable for that injury.
This means that plaintiff can recover the entire amount of his damages from any one defendant.
The doctrine applies even though each tortfeasor acted entirely independently and at different times.
Here, both the snowboarder and the employee
of the ski resort breached their duty to the skier to act with reasonable care. Each tortfeasor’s act
was the actual cause of the skier’s disability because but for either one of the acts, his leg would not have been permanently disabled. The snowboarder’s act was the proximate cause of the skier’s disability because the disability was the direct result of the snowboarder’s act. The fact that the extent of the harm was unforeseeable is irrelevant; i.e., the tortfeasor takes the victim as he finds him. Thus, the skier can recover the entire $2 million from the snowboarder.
(A) is incorrect because the contribution rules govern only whether a defendant required to pay more than his share of damages has a claim against the other jointly liable parties for the excess. Contribution does not involve the amount of damages that the plaintiff can collect in the first place.
A fire broke out in a home that had been
recently remodeled, destroying the house and
injuring the homeowner. An investigation by the
fire marshal established that the fire started from
a short in some wiring behind a wall. A small
section of wiring that ran to an outlet through a
narrow gap between a furnace chimney and a
hot water pipe had had part of its outer sheath
cut off. The homeowner filed suit against the
electrical company that did the rough wiring.
The parties stipulated for trial that the
company had installed the wiring in compliance
with the blueprints, and that the wiring had been
inspected and approved by the building inspector before the chimney and the water pipe had been installed and the walls put up, all by different contractors. At trial, the homeowner introduced the report of the fire marshal establishing how the fire started, and evidence of his medical expenses and other damages. At the end of the homeowner’s case, the electrical company’s attorney rested her case and moved for a directed verdict. The homeowner’s attorney also moved for a directed verdict.
_How should the court rule on the directed
verdict motions?_
(A) Deny the electrical company’s motion and
grant the homeowner’s motion for a directed
verdict, because a short in the wiring
caused the homeowner’s injuries.
(B) Deny the electrical company’s motion
and grant the homeowner’s motion for a
directed verdict, because the company
failed to rebut the presumption of negligence
that the homeowner has established.
(C) Deny the homeowner’s motion and grant
the electrical company’s motion for a
directed verdict, because the wire could
have been damaged by another contractor.
(D) Deny both directed verdict motions, because
the homeowner has presented enough
evidence to submit the case to the jury.
(C) The court should grant the electrical company’s motion for a directed verdict because the homeowner has not established a prima facie case of negligence on the company’s part.
The homeowner has established that the electrical company owed a duty to him and that he has suffered harm from the fire caused by the short in the wiring. However, he has not established that the company breached any duty to him.
While breach of duty is ordinarily a question for the trier of fact, a plaintiff’s failure to offer any evidence on that element of the prima facie case will permit a directed verdict for the defendant. Under certain circumstances, the fact that a particular injury occurred may itself establish or tend to establish a breach of duty owed, permitting the trier of fact to infer the defendant’s liability. This is the doctrine of res ipsa loquitur (“the thing speaks for itself”). However, for the doctrine to apply, the plaintiff must show that
(i) the accident causing his injury is the type that would not normally occur unless someone was negligent;
(ii) the negligence was attributable to the defendant; and
(iii) the injury was not attributable to the plaintiff.
The second requirement can often be satisfied by showing that the instrumentality causing the
injury was in the exclusive control of the defendant.
Here, however, the wiring was exposed to
work done by other contractors in installing a chimney and a hot water pipe nearby and putting
up the walls, and the homeowner has offered no evidence that the cut in the outer sheath of
the wiring was present when the electrical company finished its work. Instead, the fact that the wiring had been approved by the building inspector suggests that the wiring was intact when the electrical company finished. Given these facts, the homeowner has not presented evidence that the negligence was attributable to the defendant. Since res ipsa loquitur does not apply and no other evidence of breach of duty was established, the electrical company’s motion for a directed verdict should be granted.
A worker at a petrochemical plant was
severely burned when a pipe carrying hot oil
exploded. The worker brought a negligence
action against the company that manufactured
and installed the pipe. At trial, the worker
established what happened and the injuries he
suffered. He also presented evidence that the
pipe burst because it had corroded at a higher
than normal rate, which according to testimony
of the worker’s experts indicated a defect in
the manufacture of the pipe. At the close of the
worker’s case, the manufacturer moved for a
directed verdict.
How should the court rule?
(A) Deny the motion, because the pipe was
defective and injured the worker.
(B) Deny the motion, because the jury could
find that the premature corrosion of the pipe
would not have occurred absent negligence
by the manufacturer.
(B) The court should deny the motion because the jury may draw an inference of negligence from the plaintiff’s evidence. The plaintiff’s action against the manufacturer is a products liability action based on a negligence theory. In such a case, the prima facie case consists of: (i) a legal duty owed by the defendant to this plaintiff; (ii) breach of the duty; (iii) actual and proximate cause; and (iv) damages. Breach of duty requires showing (i) negligent conduct by the defendant leading to (ii) the supplying of a defective product by the defendant. The plaintiff may invoke res ipsa loquitur against the manufacturer if the error is something that usually does not occur without the negligence of the manufacturer.
Here, the plaintiff has presented evidence that the manufacturer supplied a pipe that was so defective as to be unreasonably dangerous (because of its premature corrosion). The plaintiff can use res ipsa loquitur to show negligence because the manufacturer fabricated and installed the pipe and the premature corrosion would not likely have occurred without negligence on its part. Because the plaintiff has presented evidence of the other elements
of the prima facie case, it should withstand the defendant’s motion for directed verdict.
(A) is incorrect because it implies liability without fault. As a plaintiff in a negligence action, the plaintiff must show that the manufacturer breached a duty owed to him, and that such breach caused his injuries. The mere fact that a pipe manufactured by the manufacturer exploded does not satisfy this burden. It is possible that the pipe could have exploded without any fault on the part of the manufacturer.
An ordinance of a city makes it unlawful
to park a motor vehicle on a city street within
10 feet of a fire hydrant. At 1:55 p.m., a man,
realizing he must be in the bank before it closed
at 2 p.m. and finding no other space available,
parked his automobile in front of a fire hydrant
on a city street. The man then hurried into the
bank, leaving his elderly neighbor as a passenger
in the rear seat of the car. About five minutes
later, and while the man was still in the bank, a
woman was driving down the street. The woman
swerved to avoid what she mistakenly thought
was a hole in the street and sideswiped the man’s
car. The man’s car was turned over on top of
the hydrant, breaking the hydrant and causing a
small flood of water. The man’s car was severely
damaged and the elderly neighbor was badly
injured.
If the elderly neighbor asserts a claim against
the man, will the elderly neighbor recover?
(A) Yes, because the man’s action was negligence
per se.
(C) No, because a reasonably prudent person
could not foresee injury to the elderly
neighbor as a result of the man’s action.
(C) The elderly neighbor will not recover because the man did not breach a duty owed to him when
he parked the car. This question can be analyzed in terms of either the extent of the duty of care
or proximate cause.
Where a defendant’s conduct creates an unreasonable risk of injury to persons in the position of the plaintiff, the general duty of care extends from the defendant to the plaintiff. However, no duty is imposed on a person to take precautions against events that cannot reasonably be foreseen.
And in terms of proximate cause, intervening forces that produce unforeseeable results (results not within the increased risk created by the defendant’s negligence) will be deemed to be unforeseeable and superseding, and thus break the causal connection between the defendant’s negligent act and the ultimate injury.
_Here, the man’s allegedly negligent parking
did not increase the risk that the woman would sideswipe his car; the woman’s conduct was an
unforeseeable intervening force that cuts off the man’s liability for his conduct._
(A) is incorrect for two reasons. For breach of a statute to establish negligence per se, the plaintiff must show that the statute was designed to prevent the type of harm that occurred, which does not seem to be the case with the fire hydrant ordinance here. Furthermore, proving breach of an applicable statute establishes only duty and breach of duty. Actual and proximate cause must still be established for recovery, and proximate cause is lacking here.
A husband was on his way to meet his wife
for lunch at the restaurant in the lobby of a bank
building where she worked. He had just entered
the building, which was owned and operated by
the bank, when he heard screams and the sound
of breaking glass from the restaurant area. He
immediately saw that a large piece of artwork
made of stained glass had fallen onto the seating
area of the restaurant. In the seating area he
saw several injured persons, including his wife,
lying in the wreckage of the artwork. He fainted
and hit his head on the marble floor, fracturing
his skull. The artwork had collapsed because
the pedestal that the bank had provided for the
artwork was not properly constructed.
If the husband sues the bank for his injury, is
he likely to prevail?
(B) No, because he did not actually see the
artwork collapse onto the diners.
(C) Yes, because his wife was one of the
persons he saw lying in the wreckage.
(C) The husband will recover for his injuries because his wife was among those injured by the collapse of the artwork.
The duty to avoid negligent infliction of emotional distress may be breached when the defendant creates a foreseeable risk of physical injury to the plaintiff.
In most jurisdictions, a bystander who sees the defendant negligently injuring another can recover for his own distress if
(i) the plaintiff and the person injured by the defendant’s negligence are closely related,
(ii) the plaintiff was present at the scene of the injury, and
(iii) the plaintiff personally observed or
perceived the event.
Observation is typically by sight, but may also be by hearing or other senses under certain circumstances.
Here, the husband heard the screams and the sound of breaking glass when the artwork collapsed as he entered the lobby.* *Even though he evidently did not see the artwork collapse on the diners, he heard it crash where his wife was sitting and saw the immediate aftermath. Because his wife was one of the persons injured by the collapse of the artwork, he can
recover damages for the injuries caused by his distress.
(
Common Foreseeable Intervening Forces
The following intervening forces that are normal responses or reactions to the situation created by defendant’s negligent act are almost always foreseeable:
(i) subsequent medical malpractice,
(ii) negligence of rescuers,
(iii) efforts to protect the person or property of oneself or another,
(iv) injuries caused by another
“reacting” to defendant’s actions,
(v) subsequent diseases caused by a weakened
condition, and
(vi) subsequent accident substantially caused by the original injury.
Independent Intervening Forces
Independent intervening forces that are not just a natural response or reaction to the situation created by defendant’s conduct may be foreseeable if defendant’s negligence increased the risk of harm from these forces.
Independent intervening forces include
(i) negligent acts of third persons,
(ii) crimes and intentional torts of third persons, and
(iii) acts of God.
The extent of the liability of owners and/or occupiers of land (and those in privity with
the owner/occupier) depends on where the injury occurred and on the status of the
plaintiff
1) Duty of Possessor to Those Off Premises
There is no duty to protect one off the premises from natural conditions on the premises; however, there is a duty for unreasonably dangerous artificial conditions or structures abutting adjacent land. Also, one must carry on activities on the premises so as to avoid unreasonable risk of harm to others off the premises.
Exam Tip: In urban areas, the owner/occupier is liable for damage caused off the premises by trees on the premises (e.g., falling branches). This has been an exam favorite in recent years.
2) Duty of Possessor to Those On Premises
Under the traditional rule followed in many states, the duty owed a plaintiff on the premises for dangerous conditions on the land depends on the plaintiff’s status as trespasser, licensee, or invitee.
Note, though, that close to half of the states reject the distinction between licensees and invitees (and, in a few states, trespassers as
well), and simply apply a reasonable person standard to dangerous conditions on
the land.
a) Duty Owed to Trespassers
No duty is owed to an undiscovered trespasser. As to discovered or anticipated trespassers, the landowner must:
(i) warn of or make safe concealed,
unsafe, artificial conditions known to the landowner involving risk of death
or serious bodily harm, and
(ii) use reasonable care in the exercise of “active
operations” on the property. (No duty is owed for natural conditions or less dangerous artificial conditions.)
Easement and license holders owe a duty of
reasonable care to trespassers.
b) Attractive Nuisance Doctrine
Most courts impose on a landowner the duty to exercise ordinary care to avoid a reasonably foreseeable risk of harm to children caused by dangerous conditions on his property. This is typically an artificial condition but occasionally a natural condition may suffice. To establish the doctrine’s applicability, plaintiff must show:
(i) a dangerous condition on the land that the
owner is or should be aware of,
(ii) the owner knows or should know children
frequent the vicinity of the condition,
(iii) the condition is likely to cause injury (i.e., it is dangerous because of the child’s inability to appreciate the risk), and
(iv) the expense of remedying the situation is slight compared with the magnitude of the risk.
c) Duty Owed to Licensees
A licensee is one who enters onto the land with the possessor’s permission for her own purpose or business, rather than for the possessor’s benefit. The possessor has a duty to
(i) warn of or make safe dangerous conditions (natural or artificial) known to the owner that create an unreasonable risk of harm to the licensee and that the licensee is unlikely to discover, and
(ii) exercise reasonable care in the conduct of “active operations” on the property. The possessor has no duty to inspect or repair. (Remember: Social guests are considered licensees.)
d) Duty Owed to Invitees
Invitees enter onto the land in response to an invitation by the landowner (i.e., they enter for a purpose connected with the business of the landowner or enter as members of the public for a purpose for which the land is held open to the public).
The landowner or occupier owes the same duties owed to licensees plus a duty to make reasonable inspections to discover nonobvious
dangerous conditions and, thereafter, make them safe (a warning may suffice).
One will lose invitee status if she exceeds the scope of the invitation.
e) Duty Owed to Users of Recreational Land
A landowner who permits the general public to use his land for recreational purposes without charging a fee is not liable for injuries suffered by a recreational user, unless the landowner willfully and maliciously failed to guard against or warn of a dangerous condition or activity.
3) Duties of Lessor and Lessee of Realty
The lessee has a general duty to maintain the premises. The lessor must warn of existing defects of which he is aware or has reason to know, and which he knows the lessee is not likely to discover on a reasonable inspection.
If the lessor covenants to repair, he is liable for unreasonably dangerous conditions.
If the lessor volunteers to repair and does so negligently, he is liable.
4) Duties of Vendor of Realty
A vendor must disclose to the vendee concealed, unreasonably dangerous conditions of which the vendor knows or has reason to know, and which he knows the vendee is not likely to discover on a reasonable inspection.
A 13-year-old boy who lived on a farm with
his parents in a rural area had learned to drive
the family’s tractor when he was 11. A state
statute permitted persons without a driver’s
license to operate farm vehicles on public roads
for short distances. One morning the boy took
the tractor onto a public road to reach one of the
outlying fields a few hundred yards away. As
he neared the field he was distracted by a girl
riding by on a bicycle, and cut in front of a milk
delivery truck that was starting to pass him. The
truck swerved off the road, injuring the driver.
If the driver sues the boy to recover damages
for his injuries, which of the following statements
is most correct regarding the standard of
care to be applied?
(A) The state statute replaces the general common
law standard of care with a statutory
standard.
(B) The trier of fact should take into account
the boy’s experience at driving a tractor
when considering the applicable standard of
care.
(B) The most correct statement is that the trier of fact should take into account the boy’s experience when considering the applicable standard of care.
Regardless of the specific standard of care that
is applied, someone with knowledge superior to that of the average person is required to use that knowledge. Hence, the trier of fact should take into account the fact that the boy had driven a tractor since he was 11 years old.
(A) is incorrect.
The precise standard of care in a common law negligence case may be established by proving the applicability to that case of a statute providing for criminal penalties, so that the statute’s specific duty will replace the more general common law duty of due care, and a violation of the statute will establish duty and breach of duty.
Here, nothing in the facts indicates that the boy violated any provisions in the statute, but he still may be liable to the truck driver for breach of a general duty of care.
A boy mowing his lawn noticed a strong
vibration from the engine but continued to mow.
The engine housing suddenly broke apart and
pieces flew off the lawnmower. One piece struck
the boy in the head, seriously injuring him. The
boy’s mother was inside the house and heard
yelling from the backyard. She went to the
window and saw her son lying on the ground
by the lawnmower and a friend of his kneeling
over him. She became very upset and fainted.
Subsequent investigation showed that a negligent
repair by a local mechanic caused the engine
housing to shatter. The mother brought a lawsuit
against the mechanic, seeking recovery for
her son’s injury and the emotional distress she
suffered.
Can the mother recover damages for her
emotional distress?
(A) No, because her son’s continuing to mow
after noticing the vibration was a superseding
cause of the harm.
(B) No, because the mother was not within the
zone of danger from the mechanic’s negligence.
(C) Yes, because the mother was closely related
to someone in the zone of danger from the
mechanic’s negligence.
(B) The mother cannot recover damages for her emotional distress.
A duty to avoid negligent infliction of emotional distress may be breached when the defendant creates a foreseeable risk of physical injury to the plaintiff.
The mother might try to assert two theories in support of her emotional distress claim, but she is unlikely to prevail on either one.
First, she could claim distress flowing from fear for her own safety, but she may prevail only if the defendant’s negligence placed her in a zone of danger. Here, because she was safely inside her home and quite distant from the mower when it exploded, she was not in a zone of danger.
Alternatively, she could claim distress flowing from her anguish at seeing her son injured. However, for a bystander who is outside the
zone of danger from the risk of physical injury but who suffers emotional distress from seeing the
defendant negligently injure another, most states allow recovery only if:
(i) the plaintiff and the person injured by the defendant are closely related;
(ii) the plaintiff was present at the scene of the
injury; and
(iii) the plaintiff personally observed or perceived the event.
Here, while she is related to her son, who was injured by the mechanic’s negligence, she was not present at the scene of the injury and did not personally observe or perceive the event. Hence, she cannot recover damages for negligent infliction of emotional distress.
(A) is incorrect. Under proximate cause rules, a
third party’s negligence that contributes to the plaintiff’s harm will not be considered a superseding force where it is within the foreseeable risk created by the defendant’s negligence. While the son may have been negligent in continuing to use the mower, this conduct would not cut off the defendant’s liability to the mother if she otherwise could recover for her distress.
(C) is incorrect because the plaintiff’s close relationship with the injured person is only one of the requirements for a plaintiff outside the zone of danger to recover emotional distress damages. Because the mother was not present at the scene and did not observe the event, she cannot recover.
Duty Regarding Negligent Infliction of Emotional Distress
The duty to avoid negligent infliction of emotional distress may be breached when the defendant
creates a foreseeable risk of physical injury to the plaintiff.
The plaintiff usually must satisfy two requirements to prevail:
- *(i) the plaintiff must be within the “zone of danger”; and
(ii) the plaintiff must suffer physical symptoms from the distress.**
a. Plaintiff Must Be Within the “Zone of Danger”
The plaintiff usually must show that her distress has been caused by a threat of physical impact.
b. Plaintiff Must Suffer Physical Symptoms from the Distress
Most courts usually require that the defendant’s conduct cause the plaintiff emotional distress that manifests itself in physical symptoms (note that severe shock to the nervous system that causes physical symptoms will satisfy this requirement). A growing minority of states have dropped the requirement of physical symptoms.
c. Special Situations Where Requirements Not Always Necessary
1) Bystander Not in Zone of Danger Seeing Injury to Another
A bystander outside the “zone of danger” of physical injury who sees the defendant negligently injuring another can recover damages for her own distress as long as
(i) the plaintiff and the person injured by the defendant are closely related,
(ii) the plaintiff was present at the scene of the injury, and
(iii) the plaintiff personally observed or perceived the event.
Many of these states also drop the requirement of physical symptoms in this situation.
2) Special Relationship Between Plaintiff and Defendant
The defendant may be liable for directly causing the plaintiff severe emotional distress when a duty arises from the relationship between the plaintiff and the defendant, such that the defendant’s negligence has great potential to cause emotional distress (e.g., doctor’s misdiagnosis that patient has terminal illness).
Many states drop the requirement of physical symptoms in this situation as well.
3) Other Situations
The plaintiff may be able to recover without proving the zone of danger and physical symptoms requirements in other situations where the defendant’s negligence creates a great likelihood of emotional distress (e.g., erroneous report of a relative’s death or mishandling of a relative’s corpse).
Intentional Infliction of Emotional Distress
Elements of the prima facie case:
(i) An act by defendant amounting to extreme and outrageous conduct;
(ii) Intent or recklessness;
(iii) Causation; and
(iv) Damages—severe emotional distress.
a. Extreme and Outrageous Conduct
This is conduct that transcends all bounds of decency. Conduct that is not normally outrageous may become so if:
1) It is continuous in nature;
2) It is directed toward a certain type of plaintiff(children, elderly persons, pregnant women, supersensitive adults if the sensitivities are known to defendant); or
3) It is committed by a certain type of defendant (common carriers or innkeepers
may be liable even for mere “gross insults”).
b. Requisite Intent
Unlike for other intentional torts, recklessness as to the effect of defendant’s conduct
will satisfy the intent requirement.
c. Damages
Actual damages (severe emotional distress), not nominal damages, are required. Proof of physical injury is not required. The more outrageous the conduct, the less proof of damages is required.
d. Causation in Bystander Cases
When the defendant intentionally causes physical harm to a third person and the plaintiff suffers severe emotional distress because of it, the plaintiff may recover by showing either the prima facie case elements of emotional distress or that (i) she was present when the injury occurred, (ii) she is a close relative of the injured person, and (iii) the defendant knew facts (i) and (ii).
Exam Tip Intentional infliction of emotional distress is a fallback tort position. Thus,
if another alternative in your exam question is a tort that will also allow plaintiff to
recover, it should be chosen over this alternative.
A shopper at a grocery store slipped and fell
when he stepped in some water that had seeped
out from a malfunctioning freezer case. The
fall caused the shopper to break an ankle, so
he filed suit against the store in a jurisdiction
applying the traditional rules for landowners
and possessors of land. At trial, the shopper
presented evidence of the above facts, and testified
that the floor around the water appeared
dirty.
To survive a motion for summary judgment
by the store, what additional evidence must the
shopper present?
(A) No additional evidence.
(C) The store employees knew that the freezer
case was leaking.
(A) The shopper’s lawsuit will survive a motion for summary judgment by the store without any
additional evidence.
Under the facts here, the shopper was an invitee as to the store because he came onto the premises for a purpose connected with the store’s business. _The store therefore owed him the duty to warn of nonobvious dangerous conditions and to make reasonable inspections
to discover dangerous conditions and make them safe._The shopper’s testimony that the floor
around the water appeared dirty suggests that the floor had not been swept or mopped for some time. This is enough evidence to allow the jury to decide whether the store employees failed to reasonably inspect or make safe an area in which its invitees would walk, which would breach its duty to the shopper.
(C) is incorrect because the store could be liable even if its employees did not know that water was leaking onto the floor. Because the shopper was an invitee, the store owed a duty to make reasonable inspections to discover unsafe
conditions.
A tenant remained in possession of the
house she was renting after her lease term had
expired, prompting the landlord to begin eviction
proceedings. While the tenant was still in the
house, a heavy snowfall covered the driveway,
requiring her to shovel the driveway so she could
get her car out of the garage. Shortly after she
finished shoveling, the tenant’s neighbor used
a snowblower to blow all of the snow from his
driveway onto the tenant’s driveway. Consequently,
the tenant had to shovel it again before
she could get her car out.
Which party may bring a trespass action
against the neighbor?
(A) Just the landlord, because the tenant no
longer had a right to possession of the
property.
(B) Just the tenant, because the neighbor blew
the snow onto her driveway.
(B) Only the tenant may bring a trespass action against the neighbor.
Trespass to land is an interference with the right of possession of the land. It requires
(i) an act of physical invasion of the plaintiff’s real property by the defendant, (ii) intent on the defendant’s part to bring about the
physical invasion, and (iii) causation.
Here, the tenant can allege that the neighbor knowingly blew all of the snow from his driveway onto the tenant’s driveway, causing an invasion of the tenant’s possession of her property.
(A) and is incorrect because an action for trespass may be maintained by anyone in actual or constructive possession of the land, even if that possession is without title or legal right. One who is not in possession, however, generally has no right to sue for trespass, and the landlord would not have a right to sue here because the tenant is in actual possession
of the land. While some modern cases allow a landlord to bring a damages action against
a trespasser for injury to the landlord’s interest (i.e., for any permanent damage to the property),
there was no such injury in this case. Hence, the landlord is not entitled to sue for trespass because he has suffered no interference with his right to possession.
A married couple was leaving a nightclub at
closing. The wife forgot her jacket and went back
in to the club to retrieve it while her husband
looked for a cab outside. Her husband saw a cab
sitting across the street and ran for it, cutting
off one of the club’s bouncers who was also
looking for a ride home. The bouncer became
angry at the husband for “stealing his cab.” The
cab departed while they were arguing. Seeing
no one else around, the bouncer began to punch
and kick the husband, causing him severe injury.
The wife watched the entire episode from across
the street and became greatly distressed. The
husband sued the bouncer for his injuries.
If the wife also sues the bouncer, alleging
intentional infliction of emotional distress, will
she recover?
(A) No, because the bouncer did not intend to
inflict emotional distress on the wife.
(B) No, because the bouncer did not know that
the wife was watching from across the
street.
(B) Because the bouncer was unaware that the wife was watching, the bouncer could not have the requisite intent to inflict emotional distress.
For intentional infliction of emotional distress based on conduct directed at a third person, recovery is ordinarily limited to plaintiffs who are not only present at the time, but are known by the defendant to be present, so that the mental distress is likely to have been anticipated by the defendant.
(A) is not as good an answer, because _the bouncer need not intend to inflict emotional distress. Liability for this tort may be based on
recklessness._
A 16-year-old teenager was playing baseball
in a sandlot when the ball was hit over his
head and onto a landowner’s adjacent property.
Ignoring “beware of dog” signs, the teenager
climbed over the fence into the landowner’s yard
to retrieve the ball and was attacked by a vicious
guard dog belonging to the landowner. The dog
bit the teenager, causing him to suffer severe
lacerations that required numerous stitches.
If the teenager brings an action against the
landowner to recover damages for his injuries,
will he likely prevail?
(A) Yes, because the landowner may not use a
vicious dog to protect only his property.
(B) Yes, because the landowner is strictly liable
for injuries caused by the vicious dog.
(C) No, because the teenager was trespassing
on the landowner’s property.
(A) The teenager will prevail because the landowner may not intentionally use a vicious dog to protect only his property.
One may use only reasonable force to defend property. A landowner may not use force that will cause death or serious bodily harm. Furthermore, one may not use indirect deadly force such as a trap, spring gun, or vicious dog when such force could not lawfully be
directly used, e.g., against a mere trespasser.
(B) is incorrect because strict liability in such cases generally is not imposed in favor of undiscovered trespassers against landowners. Trespassers cannot recover for injuries inflicted by the landowner’s abnormally dangerous domestic animals in the absence of negligence. (C) is incorrect because a landowner who protects his property from intruders by keeping a vicious watchdog he knows is likely to cause serious bodily harm may be liable even to trespassers for injuries caused by the animal. The liability is based on intentional tort principles: Because the landowner is not entitled to use deadly force in person to protect only property, he also may not use such force indirectly.
A fashion student at a prestigious fashion
design school bought a new sewing machine for $1,000 so that she would be more than adequately equipped for her design assignments.
One day, her roommate loaned the sewing machine to their neighbor, as she had done on several prior occasions. Unfortunately, the neighbor caused extensive damage to the machine. The cost to repair the sewing machine
was $400. If the fashion student sues her roommate for the damage the neighbor caused to the sewing machine, what will be the result?
(A) The fashion student will recover $1,000.
(B) The fashion student will recover the fair
market value of the sewing machine.
(C) The fashion student will recover $400.
(D) The fashion student will recover nothing,
because her roommate did not damage the
machine and the neighbor’s conduct was
not intentional.
(B) The fashion student will recover the fair market value of the machine because her roommate is liable for conversion.
Conversion is the intentional interference with the plaintiff’s right of possession in the chattel that is serious enough to warrant that the defendant pay the full value of the chattel. Conversion will be found if the defendant was using the chattel without permission and it was accidentally damaged, as in this case. _The remedy for conversion is the fair market value
of the chattel at the time and place of conversion._
A young teenager pointed a squirt gun at an
older teenager as if she was going to squirt him,
although the younger teenager knew that the gun was empty. The older teenager did not know that the gun was empty and yelled, “A little water
isn’t going to hurt me.” The younger teenager
pulled the trigger and yelled back, “You’re lucky,
it wasn’t even loaded.” Is the younger teenager liable to the older teenager?
(B) Yes, because the younger teenager
committed an assault.
(D) No, because the older teen was not worried
about getting wet.
(B) The younger teenager is liable for assault.
An assault is an affirmative act by the defendant
done with the intent to place the plaintiff in apprehension of an imminent harmful or offensive contact to his person and that actually causes the plaintiff apprehension. Children are deemed to be capable of forming the intent for assault and other intentional torts. The plaintiff need not be placed in fear of the contact; an apprehension of contact that is offensive (that is, not consented to) is sufficient. Furthermore, the apparent ability to inflict the contact is all that is needed; the fact that it could not be carried out is irrelevant.
Here, the older teenager was placed in apprehension of imminent offensive contact, namely the squirts of water. The younger teenager’s actions show that she intended to cause the older teenager to believe that he was going to get squirted, thus satisfying the intent requirement.
(D) is incorrect.
_While the older teenager claimed (probably truthfully) that he was not afraid of getting wet,
apprehension of offensive contact is all that the tort requires._This element is satisfied here.
A driver in the local racing circuit brought his
customized yellow stock car to a body shop to
have it repainted before the new racing season
began. When the driver returned to pick up
the car, he was horrified to discover that it was
repainted pink instead of yellow. The owner of
the body shop apologized and offered to repaint
the car, but the driver refused because the first
race was in two days. The driver lost a couple of
endorsements because the endorsers’ ads did not
work with the new color. He was also subjected
to ridicule at the track, but he felt better after he
drove the car to victory in the first race.
If the driver sues the body shop for their treatment
of his car, will he prevail?
(B) No, unless he can prove that the body shop
breached a duty of care owed to him.
(C) Yes, because the value of his car was
reduced.
(C) The driver can recover for trespass to chattels because he can show that the value of his car has been reduced as a result of the conduct of the body shop.
Trespass to chattels requires
(i) an act of defendant that interferes with plaintiff’s right of possession in the chattel,
(ii) intent to perform the act bringing about the interference with plaintiff’s right of possession,
(iii) causation, and (iv) damages.
The act of interference may be either dispossession of or damage to the chattel. Here, the body shop employees interfered with the driver’s possession of his car by painting it contrary to his instructions, and they intended to do the act (painting) that caused the interference. The driver suffered damage because that conduct reduced the value of his car for advertising purposes. Hence, the driver will be able to satisfy the prima facie case for trespass to chattels.
For false imprisonment, the plaintiff must show
(i) an act or omission on the part of the defendant that confines or restrains the plaintiff to a bounded area,
(ii) intent on the part of the defendant to
confine or restrain the plaintiff, and
(iii) causation.
A dog owner lived next door to a day care
center. Because he had a large yard and there
were no applicable zoning restrictions, he
installed a kennel and began training attack dogs
to sell to businesses. As soon as he opened the
business and posted signs in front advertising the
exceptional ferocity of the dogs, some parents
who had children enrolled in the day care center
became alarmed at the prospect of the dogs
right next to the yard where the children played,
especially because the children could see and
hear the dogs being taught to attack people.
Within a few months of the dogs’ arrival next
door, the owner of the day care lost 10% of her
enrollment.
If the day care owner brings a nuisance action
against the dog owner, what will be the most
critical factual issue that the trier of fact must
resolve to determine who should prevail?
(A) Whether the day care owner suffered
other damages in addition to her economic
losses.
(B) Whether the day care owner’s use of her
property makes her business abnormally
sensitive to the presence of the dogs.
(C) Whether the dog owner conducted his
business with reasonable care.
(D) Whether the dog owner was apprised of the
day care owner’s conc
(B) The determining factor for the day care owner in prevailing will be whether her use of the
property is abnormally sensitive to the presence of the dogs. Nuisance is an invasion of private
property rights by conduct that is either intentional, negligent, or subject to strict liability. Strict
liability will be the basis for a nuisance action (sometimes called an “absolute” nuisance or a
“nuisance per se”) when wild animals or abnormally dangerous domestic animals are involved,
or when defendant is engaged in an abnormally dangerous activity. Thus, dogs known by their
owner to be vicious may create a private nuisance when they interfere with the use and enjoyment
of the land next door, and the owner may be subject to strict liability because of his knowledge
of the dogs’ dangerous propensities. [See Restatement (Second) of Torts §822, comment
j] For the presence of the dogs to be an actionable nuisance, however, they must result in a
substantial interference with the day care owner’s use of her land. The interference will not
be characterized as substantial if it is merely the result of plaintiff’s specialized use of her own
property. [See Foster v. Preston Mill Co., 268 P.2d 645 (1954)—D not strictly liable for blasting
operations that caused female mink on P’s ranch to kill their young in reaction to the vibrations]
Hence, (B) states the most critical factual issue. (A) is incorrect because the day care owner
does not need to establish other types of damages to recover once she has established that the
dog owner’s activity is an actionable interference with the use and enjoyment of her land. (C)
is incorrect because the exercise of reasonable care by the dog owner is irrelevant; the day care
owner’s nuisance action arises from an activity for which the dog owner is strictly liable. (D) is
incorrect because the dog owner’s knowledge of his interference with the day care owner’s use of
her property would only establish that his conduct might also be an intentional nuisance, which
would require the day care owner to show unreasonableness, i.e., that her injury outweighs the
utility of his conduct. She does not need to make that showing for a nuisance action based on
strict liability.
A dog owner lived next door to a day care center. Because he had a large yard and there were no applicable zoning restrictions, he installed a kennel and began training attack dogs to sell to businesses. As soon as he opened the business and posted signs in front advertising the exceptional ferocity of the dogs, some parents
who had children enrolled in the day care center
became alarmed at the prospect of the dogs
right next to the yard where the children played,
especially because the children could see and
hear the dogs being taught to attack people.
Within a few months of the dogs’ arrival next
door, the owner of the day care lost 10% of her
enrollment. If the day care owner brings a nuisance action against the dog owner, what will be the most critical factual issue that the trier of fact must resolve to determine who should prevail?
(A) Whether the day care owner suffered
other damages in addition to her economic
losses.
(B) Whether the day care owner’s use of her
property makes her business abnormally
sensitive to the presence of the dogs.
(C) Whether the dog owner conducted his
business with reasonable care.
(D) Whether the dog owner was apprised of the
day care owner’s concerns and did nothing
to alleviate them.
(B) The determining factor for the day care owner in prevailing will be whether her use of the
property is abnormally sensitive to the presence of the dogs.
Nuisance is an invasion of private property rights by conduct that is either intentional, negligent, or subject to strict liability.
- *Strict liability will be the basis for a nuisance action (sometimes called an “absolute” nuisance or a “nuisance per se”) when wild animals or abnormally dangerous domestic animals are involved,**
- *or when defendant is engaged in an abnormally dangerous activity.**
Thus, dogs known by their owner to be vicious may create a private nuisance when they interfere with the use and enjoyment of the land next door, and the owner may be subject to strict liability because of his knowledge of the dogs’ dangerous propensities.
For the presence of the dogs to be an actionable nuisance, however, they must result in a substantial interference with the day care owner’s use of her land. The interference will not
be characterized as substantial if it is merely the result of plaintiff’s specialized use of her own property. [See Foster v. Preston Mill Co., 268 P.2d 645 (1954)—D not strictly liable for blasting operations that caused female mink on P’s ranch to kill their young in reaction to the vibrations]
Hence, (B) states the most critical factual issue.
(A) is incorrect because the day care owner
does not need to establish other types of damages to recover once she has established that the dog owner’s activity is an actionable interference with the use and enjoyment of her land.
(C) is incorrect because the exercise of reasonable care by the dog owner is irrelevant; the day care owner’s nuisance action arises from an activity for which the dog owner is strictly liable.
(D) is incorrect because the dog owner’s knowledge of his interference with the day care owner’s use of her property would only establish that his conduct might also be an intentional nuisance, which would require the day care owner to show unreasonableness, i.e., that her injury outweighs the utility of his conduct. She does not need to make that showing for a nuisance action based on strict liability.
A resident living near a factory brought a private nuisance action against the factory, alleging that emissions from the factory’s smokestack were aggravating her sinus condition and interfering with the use and enjoyment of her property.
What is the most relevant factor in the factory’s
defense of the lawsuit?
(A) The plaintiff moved to her current residence
after the factory’s smokestack was
releasing that level of emissions.
(B) No other residents of that neighborhood
have complained about the factory.
(C) The factory complies with local zoning
ordinances.
(D) No particulate matter from the emissions
has landed on the plaintiff’s property.
(B) The most relevant factor in the factory’s defense is that no one else in the neighborhood has complained.
A private nuisance is a substantial, unreasonable interference with another private individual’s use or enjoyment of her property. To constitute a substantial interference, the activity must be offensive, inconvenient, or annoying to an average person in the community. It will not be characterized as substantial if it is merely the result of the plaintiff’s hypersensitivity.
Here, if no other residents of the plaintiff’s neighborhood have complained about the emissions, that indicates that the interference may not be substantial, and that the lawsuit may stem just from the plaintiff’s persensitivity caused by her sinus condition.
(A) is incorrect because the fact that the plaintiff
“came to the nuisance” would not preclude her from recovering. Even though the factory was
emitting the same level of emissions before she moved to the property, she can recover as long
as she shows a substantial and unreasonable interference.
(C) is incorrect because compliance with government authority is not a defense to a private nuisance action. The fact that the factory
complies with zoning requirements may be some evidence as to the reasonableness of the activity
but it is not determinative, nor is it as persuasive as showing that the emissions did not interfere
with anyone else’s use of their property.
(D) is incorrect because particulate matter landing on the plaintiff’s property would be necessary for establishing a trespass action but not a nuisance action.
The interference may be substantial and unreasonable even though there was no physical invasion of the plaintiff’s property.
A farmer in a rural area contracted with a power company to erect wind turbines on his property. While zoning codes permitted this use of the property, the foundation supports for the turbines were not at the depth required by a code
provision intended to ensure that large structures
could withstand hurricane-force winds. When the wind turbines became operational, the noise bothered the farmer’s neighbor, although other neighbors the same distance away were not
bothered. The neighbor, who was a commercial
beekeeper, also discovered that his bees were
producing less honey after the turbines began
operating. If the beekeeper brings a private nuisance action against the farmer, is he likely to prevail?
(A) Yes, because the noise from the wind turbines
interfered with the beekeeper’s use
and enjoyment of his land.
(B) Yes, because the wind turbines were in
violation of the zoning code.
(C) No, because the noise from the wind
turbines apparently would not disturb
a person of ordinary sensibilities in the
community.
(D) No, because the zoning code regulation was
not designed to prevent the type of harm
that the beekeeper suffered.
(C) The beekeeper will not prevail because the noise apparently would not disturb a person of
ordinary sensibilities.
For a private nuisance action to lie, the interference with the plaintiff’s use and enjoyment of his land must be substantial. This means that it must be offensive, inconvenient,
or annoying to an average person in the community. It will not be characterized as substantial if it is merely the result of the plaintiff’s hypersensitivity or specialized use of his own property.
Here, the noise from the wind turbines does not disturb other neighbors similarly situated, indicating that the plaintiff’s disturbance was based on hypersensitivity. Thus, the fact that the beekeeper is bothered by it and that it may interfere with the beekeeping operation on his property does not make it a substantial interference. Hence, the beekeeper likely will not prevail under the circumstances here.
(A) is incorrect because it is not sufficient that the farmer’s conduct interfered with the beekeeper’s use and enjoyment of his land; the interference must be both substantial and unreasonable for the beekeeper to prevail.
(B) is incorrect because the farmer’s violation of
the zoning code does not establish that an actionable nuisance is present, particularly because the regulation that was violated pertained to the stability of the structure rather than the noise that was generated.
Conversely, (D) is incorrect because the fact that the zoning code regulation was not designed to prevent excessive noise does not establish that the farmer’s activity was not a nuisance; it could constitute a nuisance even if the structures complied with the zoning code regulation.
Slander per se categories
Business of Sex Crimes causes Disease
(Business, Sex, Crime, Disease).
adverse reflection on the plaintiff’s abilities in his trade or business
unchastity of a woman (imputing serious sexual misconduct to the P)
crime involving moral turpitude
loathsome disease (e.g., a sexually transmitted disease)
CLAMS (slander “per se”):
C – Falsely accusing the P of committing a serious CRIME
L – Falsely stating that P has an existing (not former) communicable, LOATHSOME disease (e.g., a sexually transmitted disease)
A – Making a false statement that ADVERSELY reflects on P’s trade or business
M – Falsely accusing P of MORAL turpitude
S – Falsely accusing P of serious SEXUAL misconduct (male or female)
A columnist for a major metropolitan newspaper had a very antagonistic relationship with the city’s mayor. When a restaurant owned by the columnist’s family was shut down by city health inspectors, the columnist responded with a column publicizing the shutdown and asserting that it was in retaliation for his prior columns in which he had criticized the mayor. In fact, the mayor had nothing to do with the action by the city health inspectors. While the columnist had no evidence of the mayor’s involvement, he believed that there was a connection because “that’s how the city works.” Can the mayor recover against the columnist for defamation?
(A) No, because the columnist did not act with
actual malice.
(D) Yes, because the columnist should have
investigated the accuracy of his claims before publishing the column.
(A) The mayor cannot recover against the columnist because he did not act with actual malice.
A public official, such as a mayor, may not recover for defamatory words relating to his official conduct unless there is clear and convincing proof that the statement was made with actual malice, which is defined as knowledge that the statement was false or reckless disregard as to truth or falsity.
Reckless conduct is not measured by whether a reasonable person would have investigated before publishing; rather, there must be a showing that the defendant in fact (subjectively) entertained serious doubts as to the truthfulness of his publication.
Here, while the columnist had no evidence of the mayor’s involvement with the action of the health inspectors, he believed that there was a connection based on his belief as to how the city operates. Hence, he has not acted with actual malice and is not liable to the mayor for defamation.
(D) is incorrect because the fact that the columnist should have investigated the accuracy of his assertions and did not only establishes negligence on his part. As discussed above, the mayor, as a public official, must show at least reckless disregard as to truth or falsity to recover in a defamation action.
A professional athlete was involved in a
car accident after leaving his team’s stadium
following a game, and a child was killed. The
child’s parents brought a wrongful death lawsuit
against the athlete, which eventually was settled.
A reporter for a sports gossip website accurately
reported that, as part of the legal settlement,
the athlete was required to seek counseling for
alcoholism. The website reporter incorrectly
believed that she had received this information
from a press release by the athlete’s agent, but
in fact it had been supplied by an unauthorized
source, and no public information was released
of the terms of the settlement. The athlete brought a privacy action against the website reporter based on the public disclosure of private facts, but the jury rejected his claim. What is the likely explanation?
(A) The facts stated were true.
(B) The publication involved a matter of public
interest.
(C) The report was based on a judicial
proceeding.
(D) The website reporter reasonably believed
that the athlete had consented to release of
this information.
(B) If the website reporter prevails, it will be because the publication involved a matter of public interest.
The invasion of privacy action based on public disclosure of private facts requires the following elements:
(i) publication or public disclosure by defendant of private information about the plaintiff, and
(ii) the matter made public is such that its disclosure would be highly offensive to a reasonable person.
However, publication in such a case may be privileged if the matter is one of legitimate public interest, as long as it is made with actual malice (i.e., knowledge of falsity or reckless disregard for the truth).
Here, the professional athlete may very well be considered a public figure, but, even if he is not, his conduct of possibly driving from his team’s stadium while intoxicated and striking and killing a child would appear to be a matter of public interest. The facts indicate that the report was not made with actual malice, and therefore would be privileged.
(A) is incorrect because an action for public disclosure of private facts is permitted even
if the facts disclosed were true.
(C) is incorrect. While a defendant has no liability for matters of public record, nothing in the facts indicates that the terms of the settlement were part of the public record, and no public information was released about the terms of the settlement.
(D) is incorrect because, while consent is a defense to an invasion of privacy action, a reasonable mistake as to whether consent was given is not a valid defense.
A local entertainment section of a newspaper
published a story on the town’s business district,
accompanied by photos of various businesses
in the district. A minister who happened to be
walking on the sidewalk in front of an adult
bookstore when a photo was taken for the
story became very upset when he saw it in the
newspaper, because the camera angle made it
appear that he was exiting the bookstore.
If the minister sues the newspaper for invasion
of privacy and establishes the above facts, is he
likely to prevail?
(A) Yes, because the photo made it appear as if
he was exiting an adult bookstore.
(B) Yes, because the newspaper made a public
disclosure of a private fact.
(C) No, because he was on a public sidewalk
when the photo was taken.
(D) No, because he has not alleged any
economic or pecuniary damages.
(A) The minister likely will prevail because unauthorized use of his picture that falsely makes him appear to be exiting the adult bookstore would be highly offensive to a reasonable person under the circumstances and constitute a false light invasion of privacy.
To establish a prima facie case for invasion of privacy based on publication by defendant of facts placing plaintiff in a false light, the following elements must be proved:
(i) publication of facts about plaintiff by defendant placing plaintiff in a false light in the public eye; and
(ii) the “false light” must be something
that would be highly offensive to a reasonable person under the circumstances.
Here, the photo created the false impression that the minister was exiting an adult bookstore. Publication of the photo conveying this false impression of the minister’s conduct would be highly offensive to a reasonable person under the circumstances.
(B) cannot be correct because it is couched in terms of disclosure of private facts about the plaintiff.
To establish a prima facie case for invasion of privacy based on public disclosure of private facts about plaintiff, the following elements must be proved:
(i) publication or public disclosure by defendant of private information about the plaintiff; and
(ii) the matter made public is such that its disclosure would be highly offensive to a reasonable person of ordinary sensibilities.
Here, no private facts were disclosed, and therefore an action based on public disclosure of private facts will not succeed.
(C) is incorrect. Because the minister was placed in a false light, it makes no difference that the picture was taken on a public sidewalk. An invasion of privacy based on false light can occur on public property as well as private property.
(D) is incorrect. The absence of economic harm to the minister has no bearing on a privacy action. In an action for invasion of right to privacy, the plaintiff need not plead and prove special damages, providing the elements of a prima facie case are present. In other words, emotional distress and mental anguish are sufficient damages.
The owner of a small computer consulting firm was attending the annual trade meeting of
the computer industry and spoke with the owner
of a second consulting firm about doing joint
projects. The owner of the second firm replied
by rejecting the idea immediately, stating that
she believed that the first owner was incompetent. A sales representative of a computer supply firm overheard the remark. The owner of the first firm sued the owner of the second firm for defamation. If the first owner does not prevail in this lawsuit, what will be the likely reason?
(A) It was not reasonably foreseeable that the
second owner’s remark would be overheard.
(B) The second owner did not know that her
remark would be overheard.
(C) There was no publication.
(D) The sales representative was not a party to
the conversation.
(A) If the plaintiff does not prevail, it will be because it was not reasonably foreseeable that the defendant’s remark would be overheard, and therefore the fault requirement for the publication
element would not be satisfied.
To establish a prima facie case for defamation, the following elements must be proved:
(i) defamatory language on the part of the defendant;
(ii) the defamatory language must be “of or concerning” the plaintiff (i.e., it must identify the plaintiff to a reasonable reader, listener, or viewer);
(iii) publication of the defamatory language by the defendant to a third person; and
(iv) damage to the reputation of the plaintiff.
The second owner’s statement constitutes
defamatory language because it adversely affects the first owner’s reputation by attacking
his competency. The publication requirement is satisfied when there is a communication to a third
person who understands it. However, the communication to the third person must be made either intentionally or negligently; if it was not reasonably foreseeable that the defamatory statement would be overheard by the sales representative, the fault requirement for the publication element is not satisfied.
(B) is incorrect. The plaintiff could prevail even if the defendant did not know that her remark would be overheard as long as it was reasonably foreseeable that it could be overheard.
(C) is incorrect because there in fact was a publication, i.e., there was a communication, albeit not intentionally made, to the sales representative, who would reasonably have understood it to be defamatory.
(D) is incorrect because there is no requirement that the third party be a party to the
conversation—the third party need only be a reader, listener, or viewer.
To establish a prima facie case for interference with business relations, the following elements must
be proved:
(i) existence of a valid contractual relationship between plaintiff and a third party or a
valid business expectancy of plaintiff;
(ii) defendant’s knowledge of the relationship or expectancy;
(iii) intentional interference by defendant that induces a breach or termination of the relationship or expectancy; and
(iv) damage to plaintiff.
An interferer’s conduct may be privileged where it is a proper attempt to obtain business for the interferer, particularly if the interference is only with a prospective business relationship rather than with an existing contract.
Strict liability for wild animals includes….
….liability for the harm that results when a person is attempting to flee from what is perceived to be a dangerous animal.
A bull, which is…
animal.
a domestic
The driver of a tanker truck was transporting
radioactive waste from a nuclear power plant to
a permanent storage facility in a remote western
region of the United States. After driving all
night, the driver fell asleep at the wheel and the
truck crossed over the center line, off the road,
and onto a homeowner’s property, coming to rest
after crashing into several glass cases containing
the homeowner’s collection of poisonous snakes,
the keeping of which was permitted by local
ordinance. When the driver exited the truck, he
was bitten on the leg by one of the poisonous
snakes and became seriously ill. The driver brought an action against the homeowner for his injuries. The parties stipulated to the above facts, and that the driver violated a state statute by driving off of the road. Both parties moved for judgment as a matter of law on the liability issue.
How should the court rule?
(A) Grant the driver’s motion and deny the
homeowner’s motion, because the homeowner
is strictly liable for the injury caused
by the snake.
(B) Deny the driver’s motion and grant the
homeowner’s motion, because the driver
was a trespasser on the homeowner’s
property.
(C) Deny the driver’s motion and grant the
homeowner’s motion, because the driver’s
violation of the state statute establishes
contributory negligence as a matter of law.
(D) Deny both parties’ motions, because both
parties were engaged in an activity for
which strict liability is imposed.
(B) The court should grant the homeowner’s motion for judgment as a matter of law because
the driver has not established a prima facie case against the homeowner.
An owner of wild (dangerous) animals is strictly liable for injuries caused by those animals as long as the personinjured did nothing, voluntarily or consciously, to bring about the injury. However, strict liability generally is not imposed in favor of undiscovered trespassers against landowners in the absence of negligence, such as when the landowner knows that the trespassers are on the land and fails to warn them of the animal.
Here, despite the fact that the driver did not intend to enter the homeowner’s land (and thus would not be liable for the intentional tort of trespass), his status on the homeowner’s land is that of a trespasser rather than a licensee or invitee. The driver has presented no evidence of negligence on the homeowner’s part and therefore has not established a prima facie case against the homeowner.
(A) is wrong because, as discussed above, the
homeowner is not strictly liable to the driver because the driver was a trespasser.
(C) is incorrect because the driver will not prevail regardless of whether he was contributorily negligent, because he cannot establish a prima facie case against the homeowner in either negligence or strict liability.
(D) is incorrect for several reasons: While the driver’s transport of radioactive waste may have been an abnormally dangerous activity, that danger had nothing to do with the accident
that occurred. Furthermore, the fact that the driver may have been engaged in an abnormally
dangerous activity would not prevent him from recovering damages from another tortfeasor if he
established the requisite prima facie case. Finally, the fact that the parties were engaged in activities potentially creating strict liability has nothing to do with whether issues of fact regarding liability still exist that would require denying both motions and going to trial.
A fan of a popular band sneaked behind stage
to try to meet band members. She entered a
small room to hide despite a sign on the door
marked, “Danger – Electrical Equipment.”
During the performance, the band’s special
effects equipment in the room malfunctioned
and triggered fireworks in the room to explode.
The fireworks had been stored in the room by
a third party that the band had hired to stage
the special effects. The fan suffered burns to
her face and hands from the fireworks. The fan
brought an action based on strict liability against
the band. After evidence of the above facts was
presented, both parties moved for a directed
verdict.
How should the court decide on the directed
verdict motions?
(A) Grant the band’s motion, because the fan
was a trespasser and could not rely on a
strict liability theory.
(B) Grant the fan’s motion, because the judge
can determine as a matter of law that the
band’s storage and use of fireworks for
their special effects was an abnormally
dangerous activity.
(C) Deny both motions, because the jury must
determine whether the malfunction was
caused by the third party that the band
hired to set up the special effects.
(D) Deny both motions, because the jury must
determine whether the fan was a foreseeable
plaintiff.
(D) The court should deny both motions.
An activity may be characterized as abnormally dangerous if it involves a substantial risk of serious harm to persons or property even when reasonable care is exercised. Whether an activity is abnormally dangerous is a question of law that the court can decide on a motion for directed verdict. However, the defendant is liable only to “foreseeable plaintiffs”—persons to whom a reasonable person would have foreseen a risk of harm under the circumstances. Generally, strict liability is not imposed on a defendant’s abnormally dangerous activity that caused injury to a person whose presence the defendant had no reason to know.
Here, there is nothing to suggest that the band knew that the fan had sneaked into the equipment room. Hence, the jury could determine that the fan’s presence was unforeseeable and reject her strict liability claim.
(A) is incorrect because the fan’s status as a trespasser would be relevant only in an action against the landowner. In this action against the band, the issue is whether the fan is a foreseeable plaintiff, which is a question for the jury.
(B) is incorrect even though it is true that
the judge makes the determination whether an activity is abnormally dangerous. As discussed
above, even if the judge determines that the band’s storage and use of fireworks is an abnormally dangerous activity, the jury needs to determine whether the fan is a foreseeable plaintiff.
(C) is incorrect because the band would be liable even if the third party that it had hired was responsible for the harm that occurred. Regardless of whether the use of fireworks is determined to be an abnormally dangerous activity for which strict liability applies, it is clearly an inherently dangerous activity because fireworks injuries occur even in the course of professional fireworks displays. Hence, the band would be vicariously liable for any fault on the part of the third party in setting up the special effects or storing the fireworks.
To prove breach of duty in a products liability case based on negligence, the plaintiff must show:
Defective food products are treated as …
(i) negligent conduct by the defendant leading to
(ii) the supplying of a defective product by the defendant.
… manufacturing defects—the product is defective if it is dangerous beyond the expectation of the reasonable consumer.
To show negligence in a manufacturing defect case, the plaintiff may invoke res ipsa loquitur. To rely on that doctrine, the plaintiff must show that the injury is the type that would not normally occur absent negligence and that it usually occurs because of the negligence of the
manufacturer (and that the injury was not attributable to the plaintiff).
An intermediary’s failure to discover a defect is not a superseding cause, and the defendant who created the defect will remain liable.
i.e.
A piece of bone over an inch long in ground beef was a dangerous defect that would not have occurred without negligence on the part of the meat processor.The processor would be liable even if the restaurant employees were negligent in not finding the bone,
A homeowner purchased a riding lawn mower
from a lawn mower dealer. During his first use
of the mower, the homeowner noticed that the
mower was vibrating when he turned, but he
was able to finish mowing. A few days later, the
homeowner lent the mower to his neighbor. The
neighbor was driving the mower back to his yard
when he made a turn and a wheel broke off,
causing the neighbor to be thrown off the lawn
mower and onto the sidewalk. The neighbor was
injured.
The neighbor brought a negligence action
against the dealer for his injuries. At trial, the
neighbor presented evidence that the wheel
broke because of a manufacturing defect. The
dealer presented evidence that the homeowner
could have discovered the defect after the mower
began vibrating when he used it for the first time.
In this action, who is likely to prevail?
(A) The neighbor, because the lawn mower was
sold by the dealer with an unreasonably
dangerous defect.
(B) The neighbor, because the defect in the
wheel would not likely have occurred in the
absence of negligence.
(C) The dealer, because the homeowner should
have discovered the defect when the mower
first started vibrating.
(D) The dealer, because there is no evidence
that the dealer had reason to know that the
lawn mower was defective.
(D) The dealer will prevail because there is no evidence that it should have discovered the defect.
To prove breach of duty in a products liability action based on negligence, the plaintiff must show
(i) negligent conduct by the defendant leading to (ii) the supplying of a defective product by the
defendant.
_However, *a dealer* who buys from a reputable manufacturer with no reason to anticipate that the product is dangerous need make only a cursory inspection of the goods to avoid liability
for manufacturing defects._
Here, there is no evidence that the dealer should have known that the wheel was defective; hence the dealer will likely prevail.
(A) is incorrect because merely selling the lawn mower with an unreasonably dangerous defect, without knowing or being expected to know of the defect, will not subject the dealer to liability for negligence. The statement in (A) is more appropriate in an action based on strict liability.
(B) is incorrect because the use of res ipsa
loquitur suggested by that choice would be directed at the manufacturer rather than the dealer, and the dealer is not liable for the manufacturer’s negligence.
(C) is incorrect because the negligent failure of an intermediary to discover a defect is not a superseding cause. If the dealer were otherwise liable, the negligent failure of the homeowner to discover the defect in the wheel would not cut off the dealer’s liability.
A man purchased a large flat screen television and decided to mount it on the ceiling over
his bed. The manual that came with the product
included detailed instructions and illustrations
on how to mount the television on different types
of walls, along with all the required hardware,
but contained neither instructions nor warnings
regarding mounting on the ceiling. The man
carefully followed the wall-mounting instructions
and was satisfied that it would hold. In fact, however, the mounting was not appropriate for ceilings. The next night, a woman who was
the man’s overnight guest was seriously injured
when the television came loose and fell on the
bed. Will the woman prevail in a suit against the
company that manufactured the television?
(A) Yes, because the manufacturer had a duty
to include warnings for all potential placements
of its product.
(B) Yes, if the manufacturer knew that its
television was sometimes mounted on ceilings rather than walls.
(B) Knowledge on the part of the manufacturer that its television was being mounted on the ceiling would give rise to a duty to include in the manual warnings against the practice or detailed instructions on how to safely mount it.
The television hardware and instructions were appropriate for its intended mounting on the wall. However, courts in a strict liability case require a commercial supplier to anticipate reasonably foreseeable uses even if they are misuses of the product. If the manufacturer knew that members of the public were sometimes mounting the television on the ceiling, marketing the product without including either warnings against the practice or appropriate hardware and instructions on how to safely do so made the product so defective as to be unreasonably
dangerous if it were improperly mounted. Under a strict liability theory, the manufacturer is
liable for supplying a defective product. As a guest of a purchaser of the product, the woman is a foreseeable plaintiff; thus, the manufacturer may be liable to her. The defective product actually and proximately caused the woman to suffer serious injuries. Therefore, the manufacturer is liable to the woman in a strict products liability action.
(A) is incorrect because the facts do not establish that the manufacturer was under a duty to include the warnings in its manual. Such a duty would exist if the manufacturer knew (as (B) states) or should have known that the television was being mounted on ceilings.
Under either negligence or strict liability theory, an intermediary’s negligent failure to discover a defect is (not) a superseding cause,
so the defendant who supplied the defective product will still be liable.
is not
A patient was admitted to a hospital to undergo an x-ray procedure that required him to be tilted up. During the procedure, the technician improperly strapped the patient to an x-ray
table designed for the procedure. There was a footrest at the bottom of the table to help support the patient, but the bolts holding the footrest gave way and the footrest fell off. Had the patient been securely strapped in, he would have remained on the table. However, the loss of the footrest caused sufficient additional pressure
on the straps that they loosened, and the patient
fell from the table, struck his head, and suffered
injuries. The patient sued the manufacturer of
the x-ray table, which supplied the table fully
assembled, on a theory of strict liability for his
injuries. Who is likely to prevail?
(A) The patient, because the x-ray table was
dangerously defective.
(C) The manufacturer, because the hospital
employee was negligent when he failed to
properly secure the straps.
(D) The manufacturer, because the table was
not in its exclusive control when the plaintiff
incurred his injury.
(A) Judgment will likely be for the plaintiff because the manufacturer supplied a product that was in a defective condition unreasonably dangerous to users.
A commercial supplier of a product may be
liable to any foreseeable plaintiff who was injured by a product that was in a defective condition
unreasonably dangerous to users. In making this determination in case of a design defect, courts
evaluate whether a less dangerous design or alternative was economically feasible. In making
this determination in the case of a manufacturing defect, courts evaluate whether the product
was dangerous beyond the expectation of the ordinary consumer.
Here, the bolts attaching the footrest to the x-ray table gave way. The table had been fully assembled by the manufacturer and
was being used appropriately by the hospital, indicating that the failure of the bolts was due to a manufacturing or design defect in the x-ray table.
Under either the “feasible alternative” test or the “consumer expectation” test, the trier of fact will likely find that the product was defective and
unreasonably dangerous to users when the product left the manufacturer’s control, making the manufacturer strictly liable.
(C) is wrong because an intermediary’s ordinary negligence is not a superseding cause that would relieve the manufacturer from liability.
Here, the injury was caused by both the failure of the footrest and the improper tightening of the
straps. The latter occurrence is ordinary foreseeable negligence that would not supersede whatever liability the manufacturer is judged to have for the failure of the footrest.
(D) is wrong because the fact that the table was not in the exclusive control of the manufacturer is irrelevant.
Exclusive control is a means of showing an element of res ipsa loquitur, a doctrine for proving breach of duty in a negligence action. In a strict products liability action, the manufacturer will be liable for an unreasonably dangerous defective condition as long as the table was expected to, and did in fact, become operational without substantial change in the condition in which it was supplied.
A father purchased the largest model rocket
that he could find, hoping to interest his daughter
in space engineering. Several days later, the
father and daughter went to an open area in a
state park to fire the rocket. The father was in
such a rush to fire the rocket that he failed to
remove the safety key from the launch system.
Upon connecting the launch system to the rocket
motor, the motor exploded, seriously injuring the
father and daughter.
A negligence action was brought on behalf of
the daughter against the rocket manufacturer. At
trial, evidence shows that the explosion resulted
from a manufacturing defect in the motor.
Evidence also shows that the motor would not
have exploded while they were connecting the
launch system if the father had removed the
safety key.
Will the daughter recover for her injuries?
(A) No, because she did not purchase the
rocket.
(B) No, because the father was negligent in
failing to remove the safety key before
connecting the motor.
(C) Yes, if the trier of fact determines that the
defect in the motor could have been discovered
by the rocket manufacturer in the
exercise of reasonable care.
(D) Yes, unless the manufacturer shows that the
defect in the motor could have been discovered
by the rocket retailer in the exercise of
reasonable care.
(C) The rocket manufacturer will be liable for negligence if it should have discovered the defect in the motor.
_To establish a prima facie case for negligence in a products liability case, the plaintiff must show the existence of a legal duty owed by the defendant to that particular plaintiff, breach
of that duty, actual and proximate cause, and damages._
_To prove breach of duty, plaintiff must
show (i) negligent conduct by the defendant leading to (ii) the supplying of a defective product by the defendant._
The call of the question indicates that the rocket manufacturer supplied a defective product. If the defect could have been discovered by the manufacturer in the exercise of reasonable
care, it was negligent in not discovering the defect and preventing the rocket from being sold.
Choice (A), which addresses the duty element, is incorrect because a manufacturer of a defective
product owes a duty of due care to any foreseeable plaintiff. Although the daughter did not purchase the rocket (and so was not in privity with the manufacturer), she was the daughter of the purchaser and was standing nearby when her father connected the launch system and the rocket exploded, causing her injuries.
(B) is incorrect because the father’s negligence would be imputed to the daughter only where she and her father stand in such a relationship to each other (e.g., an employer-employee relationship) that the courts would find it proper to charge the daughter with her father’s negligence (such that she would be vicariously liable for her father’s negligent conduct
if a third party had sued her). Here, there are no facts to impute the father’s negligence (in failing
to remove the safety key) to his daughter.
Choice (D), which addresses the causation element, is incorrect because an intermediary’s negligent failure to discover a defect is foreseeable negligence and therefore not a superseding cause. The rocket manufacturer is the defendant whose original negligence created the defect and will still be liable.
A security guard, dressed in plain clothes, was working for a discount store when a customer got into a heated argument with a cashier over the store’s refund policy. Without identifying himself as a security guard, the security guard suddenly grabbed the customer’s arm. The customer attempted to push the security guard away, and the security guard knocked the customer to the floor, causing injuries. The customer sued the discount store for battery on a theory of vicarious liability for the injuries caused by the security guard.
The store filed an answer to the customer’s complaint, asserting the affirmative defense of contributory negligence. The customer has moved to strike the affirmative defense. Traditional rules of contributory negligence apply.
Should the trial court grant the customer’s motion?
A. No, because contributory negligence is an affirmative defense to a cause of action based on vicarious liability.
B. No, because the customer should have known that his argument with the cashier might provoke an action by a security guard.
C. Yes, because contributory negligence is not a defense to battery.
D. Yes, because the customer did not know that he was pushing away someone who was employed as a security guard.
(C) is correct.
Contributory negligence is a defense to negligence actions but not to intentional torts actions. Battery is an intentional tort for which contributory negligence is not a defense. Hence, the court should grant the customer’s motion to strike the affirmative defense.
(A) is incorrect. A cause of action based on vicarious liability may arise not just from an employee’s negligence but also from an employee’s intentional conduct in certain circumstances. Here, the vicarious liability allegation is based on the security guard’s intentional tort of battery, for which contributory negligence is not a defense.
(B) is incorrect. Even if the customer should have known that a security guard would intervene and was therefore contributorily negligent, that would not be a defense to battery.
(D) is incorrect. The fact that the customer did not know that he was pushing a security guard does not address the store’s defense, which is that the customer should have known that the person who intervened could have been a security guard and therefore was contributorily negligent. Choice (C) addresses why the defense will fail.
The tort of invasion of privacy includes the following four kinds of wrongs:
(i) appropriation by the defendant of the plaintiff’s picture or name for the defendant’s commercial advantage;
(ii) intrusion by the defendant on the plaintiff’s affairs or seclusion; (taking video of someone on a public street is not actionable)
(iii) publication by the defendant of facts placing the plaintiff in a false light; and
(iv) public disclosures of private facts about the plaintiff by the defendant (taking video of someone on a public street is not actionable)
***
the right of privacy is a personal right and does not survive the death of the plaintiff
Under contributory negligence principles, a plaintiff is required to …
act as a reasonably prudent person, the same as the defendant.
A plaintiff who fails to act reasonably and who negligently contributes to his injury will have his recovery reduced in jurisdictions following a pure comparative negligence rule.
In a case where contributory negligence is shown, the trier of fact weighs the plaintiff’s negligence against that of the defendant and reduces the plaintiff’s damages accordingly.
i.e.
The worker was in no danger in the stalled elevator but nevertheless jumped 12 feet to the ground after waiting 15 minutes. The trier of fact is likely to find that the worker was at least partly at fault for his injury by jumping from that height. Hence, he will not obtain a judgment for 100% of his damages.
A patient who had suffered a severe fracture of her leg was treated by an orthopedist, who set the patient’s leg and put it in a cast. When the leg continued to bother the patient six months later, she consulted a second orthopedist in the same town. The second orthopedist surgically inserted a pin to facilitate healing.
The patient brought a malpractice action against the first orthopedist, claiming that he should have surgically inserted a pin at the time of initial treatment.
The only evidence that the patient offered in support of her malpractice claim was the testimony of the second orthopedist, as follows:
In response to the question “Would you have inserted a pin initially?” the second orthopedist testified, “I personally would not have been satisfied that the leg would heal properly without a pin.”
At the close of the patient’s evidence, the first orthopedist moved for judgment as a matter of law.
A. No, because the patient has introduced evidence that the first orthopedist failed to give the care that the second orthopedist would have provided.
B. No, because the second orthopedist practices in the same town and field of specialty as the first orthopedist.
C. Yes, because the patient has failed to introduce evidence that the first orthopedist’s care fell below the professional standard of care.
D. Yes, because the second orthopedist also treated the patient and is thus not sufficiently objective to offer expert testimony.
Should the motion be granted?
(C) is correct. A person who is a professional (e.g., doctor) is required to possess and exercise the knowledge and skill of a member of the profession or occupation in good standing. For doctors, most courts will apply a national standard of care to evaluate their conduct. This standard of care generally is established by expert testimony.
Here, the only evidence the patient offered in support of her malpractice claim was the testimony of the second orthopedist. However, that orthopedist stated only that he personally would have inserted a pin to make sure the leg healed properly. He did not testify as to what treatment conforms to the requisite standard of care or whether the first orthopedist had failed to conform to that standard of care by not inserting a pin initially. Hence, the patient has not presented sufficient evidence of breach of duty to withstand a motion for judgment as a matter of law. (A) is incorrect. Evidence that the second orthopedist would have provided different care is not enough to establish breach of duty, as discussed above. (B) is incorrect. As discussed above, most courts apply a national standard of care to evaluate the standard of care of doctors, particularly if they are specialists. Thus, the fact that the second orthopedist practices in the same town as the first orthopedist is irrelevant. (D) is incorrect. The fact that the second orthopedist treated the patient does not preclude him from testifying to the appropriate standard of care.
A fumigation company was hired to eliminate pests in one of two buildings in a condominium complex that shared a common wall. The owners of the complex told the fumigation company that the common wall separating the infested building from the uninfested building was an impenetrable fire wall. The fumigation company did its own thorough inspection and determined that the buildings were indeed completely separated by the wall. Residents of the condominium units in the building that was to be sprayed were told to evacuate, but the residents of the uninfested building were told that they could remain while the other building was treated.
During and shortly after the fumigation, in which a highly toxic chemical was used, many residents of the uninfested building became sick. It was determined that their illnesses were caused by the fumigation chemical.
In fact, there was a hole in the fire wall separating the two buildings, but because it could only be observed from a specific position in the crawl space underneath the floor of the uninfested building, it had not been discovered by either the fumigation company or any previous building inspector.
Are the residents of the uninfested building likely to prevail in a tort action against the fumigation company?
A. No, because the condominium complex owners were responsible for accurately conveying the condition of their buildings.
B. No, because the fumigation company exercised a high level of care.
C. Yes, because the fumigation company can be held strictly liable for its activity.
D. Yes, because the fumigation company put a dangerous product into the stream of commerce.
C) is correct.
A defendant may be held strictly liable for an activity that is deemed abnormally dangerous. The courts generally impose two requirements for finding an activity to be abnormally dangerous:
(i) the activity must create a foreseeable risk of serious harm even when reasonable care is exercised by all actors; and
(ii) the activity is not a matter of common usage in the community.
Fumigating is commonly held to be an abnormally dangerous activity for which strict liability is imposed.
(A) is incorrect. The fumigation company did its own thorough inspection of the buildings and did not rely on the information from the condominium complex owners. Even had that been the case, the fumigation company would be strictly liable for the harm that occurred.
(B) is incorrect. Because the fumigation company is strictly liable for its activity, it does not matter that it exercised a high level of care.
(D) is incorrect. The fumigation company used a dangerous product but it did not put it into the stream of commerce. Hence, its strict liability is based on engaging in an abnormally dangerous activity rather than supplying a dangerous and defective product.